Anda di halaman 1dari 0

ACCOUNTING FOR MANAGERS

MBA1st SEMESTER, M.D.U., ROHTAK


SYLLABUS
External Marks : 70
Time : 3 hrs.
Internal Marks : 30
143
UNIT-I
UNIT-II
UNIT-III
UNIT-IV
Financial Accounting-concept, importance and scope, accounting
principles, journal, ledger, trial balance, depreciation (straight line and
diminishing balance methodology), preparation of final accounts with
adjustments.
Ratio analysis, fund flowanalysis, cashflowanalysis.
Management accounting-concept, need, importance and scope; cost
accounting-meaning, importance, methods, techniques and
classificationof costs, inventory valuation.
Budgetary control-meaning, need, objectives, essentials of budgeting,
different types of budgets; standard costing and variance analysis
(materials, labour); marginal costing and its application in managerial
decisionmaking.
footer
Q. Define Accounting. Explain its Nature.
:-
:
According to American Institute of Certified Public Accountants:
According to R.N. Anthony :
:
(1) Recording of Financial Transactions only :
(2) Recording :
Ans. Accounting is often called the language of business. The
basic function of any language is to communicate. Accounting communicates
the results of the business to the users of accounting information to enable
them to make effective decisions. To communicate information, accounting
follows a systematic process of recording, classifying and summarizing of
numerous business transactions resulting in creation of financial statements.
The two most important financial statements are :
(i) Trading, Profit &Loss Account.
(ii) Balance Sheet.
Accounting is the art of recording, classifying and summarizing ina significant
manner and in terms of money, transactions and events which are, in part
atleast, of a financial character, and interpreting the results thereof.
Nearly every business enterprise has accounting system. It is a means of
collecting, summarizing, analyzing and reporting in monetary terms,
informations about business.
Only those transactions
and events are recorded in accounting which can be expressed in terms of
money. Those transactions which cannot be expressed in terms of money
are not recorded in accounting like the value of human resource, strike by
employees, and change inmanagerial policies etc.
Accounting is the art of recording of business transactions
according to some specified rules. In a small business where number of
transactions is quite small, all transactions are first of all recorded in a
Accounting
Definitionof Accounting
Feature or Characteristics or Nature of Accounting
ACCOUNTING FOR MANAGERS
MBA 1st Semester (DDE)
UNIT I
144
footer
book called Journal. But in a big business where the number of
transactions is quite large, the Journal is further sub-divided into various
subsidiary books suchas:-
(i) CashBook
(ii) Purchase Book
(iii) Sales Book
(iv) Purchase ReturnBook
(v) Sales ReturnBook.
The number of subsidiary books to be maintained depends on the size and
nature of the business.
After recording the transactions in journal or subsidiary
books, the transactions are classified. Classification is the process of
grouping the transactions of one nature at one place, in a separate
account. The books in which various accounts are opened is called
Ledger.
Summarising involves the balancing of Ledger accounts
and the preparation of Trial Balance with the help of such Balances.
Financial Statements are prepared withthe help of trial balance. Financial
statements are includes:-
(i) Trading, Profit &Loss Account
(ii) Balance Sheet.
In accounting the results of business are
presented in such a manner that the parties interested in the business
such as proprietors, managers banks, creditors etc. can have full
information about the profitability and the financial position of the
business.
It refers to transmission of summarized and
interpreted informationto a variety of users. The users are:-
(i) Creditors
(ii) Investors
(iii) Lenders
(iv) Government
(v) Proprietors
(vi) Management
(vii) Banks etc.
Ans. Accounting is often called the language of business. The
basic function of any language is to communicate. Accounting communicates
(3) Classifying :
(4) Summarising :
(5) Interpretation of the Results :
(6) Communicating :
Q. Define Accounting. Also explain its Importance.
: Accounting
145
ACCOUNTING FOR MANAGERS
footer
the results of the business to the users of accounting information to enable
them to make effective decisions. To communicate information, accounting
follows a systematic process of recording, classifying and summarizing of
numerous business transactions resulting in creation of financial statements.
The two most important financial statements are:-
(i) Trading, Profit &Loss Account.
(ii) Balance Sheet.
Accounting is the art of recording, classifying and summarizing in a
significant manner and in terms of money, transactions and events which are,
inpart atleast, of a financial character, and interpreting the results thereof.
Management needs a lot of
information for the efficient running of the business. All such information
is provided by the accounting which helps the management in the
following:-
Management would like to know whether the
sales are increasing or decreasing and also the speed of increase in
the cost of production. All such information is provided by the
accounting, which helps the management in estimating the future
sales and expenses. It also helps them to estimate the cash receipts
and cashdisbursements during the next accounting period.
At times, the Management has to
take a number of decisions. Accounting provides all the informations
required for making suchdecisions.
Management would like to see that the cost
incurred is reasonable and that no department is overspending.
Accounting provides informationto the management inthis regard.
Business transactions
have grown in size and complexity and it is not possible to remember each
and every transaction. Accounting keeps a prompt and systematic record
of all the transactions and summarizes them in order to provide a true
picture of the activities of the business entity.
Accounting reports the net
result of business activities of an accounting period. For this purpose
Trading and Profit &Loss Account of the business is prepared at the end of
eachaccounting period. All the items relating to purchase, sales, expenses
and revenues (Income) of the business are recorded in Trading, Profit &
Loss Account.
Definitionof Accounting :
According to American Institute of Certified Public Accountants :
Importance of Accounting :
(1) Helpful in Management of Business :
(i) Helpful in Planning :
(ii) Helpful in Decision-Making :
(iii) Helpful in Controlling :
(2) Provides Complete and Systematic Record :
(3) Information regarding Profit or Loss :
146
footer
If Revenues >Expenses-Profit
If Revenues< Expenses-Loss
For a businessman,
merely ascertaining profit or loss of the business is not sufficient. The
businessmanmust also knowthe financial healthof the business. For this
purpose a statement called Balance Sheet is prepared which shows the
assets on the one hand and the liabilities and capital on the other hand.
Balance Sheet describe the following :
(i) Howmuchthe business has to recover fromDebtors?
(ii) Howmuchthe business has to pay to Creditors?
(iii) Howmuchthe business has inthe formof
(a) Cash-in-hand (b) Cashat Bank
(c) Closing Stock (d) Fixed Assets.
By keeping a systematic record
accounting helps the owners to compare one years costs, expenses, sales
and profit etc. with those of other years. Such a comparison provides the
useful information on the basis of which important decisions can be taken
more judiciously.
Another main objectives of
accounting is to communicate the accounting information to various
users like:
(i) Creditors
(ii) Investors
(iii) Lenders
(iv) Government
(v) Proprietors
(vi) Management
(vii) Banks etc.
Another objective of accounting is to
provide information about liquidity position. For this purpose it prepares
a Cash Flow Statement. It depicts inflows and outflows of cash from
operating, investing and financing activities.
One of the main objectives of accounting is to
provide bases for filing tax returns relating to income tax, sales tax, value
added tax, service tax, etc.
If a business entity is being sold, the
accounting information can be utilized to determine the proper purchase
price.
Accounting information is of great help while
raising loans frombanks or other financial institutions. Such institutions
(4) Information Regarding Financial Position :
(5) Enables Comparative Study :
(6) Provide Informations to Various Parties :
(7) To Know the Liquidity Position :
(8) To File Tax Returns :
(9) Facilitates Sale of Business :
(10) Helpful in Raising Loans :
147
ACCOUNTING FOR MANAGERS
footer
before sanctioning loan screen various financial statements of the firm
suchas final accounts, fund flowstatement, cashflowstatement etc.
Ans. Accounting is often called the language of business. The
basic function of any language is to communicate. Accounting communicates
the results of the business to the users of accounting information to enable
them to make effective decisions. To communicate information, accounting
follows a systematic process of recording, classifying and summarizing of
numerous business transactions resulting in creation of financial statements.
The two most important financial statements are:-
(i) Trading, Profit &Loss Account.
(ii) Balance Sheet.
Accounting is the art of recording, classifying and summarizing ina significant
manner and in terms of money, transactions and events which are, in part
atleast, of a financial character, and interpreting the results thereof.
In order to appreciate the exact nature and scope of
accounting, we must understand the following aspects of accounting:
Accounting records only economic events. An
economic event is a transaction which can be measured and expressed in
terms of money.
It means determining what transactions are to be
recorded. It involves observing events and selecting those events that are
of financial character and relate to the organization.
It means quantification of business transactions into
financial terms by using monetary units.
Accounting is the art of recording of business transactions
according to some specified rules. In a small business where number of
transactions is quite small, all transactions are first of all recorded in a
book called Journal. But in a big business where the number of
transactions is quite large, the Journal is further sub-divided into various
subsidiary books suchas:-
CashBook
Purchase Book
Sales Book
Purchase ReturnBook
Sales ReturnBook.
(11) Helpful inPrevention and Detectionof Errors and Frauds.
Q. Define Accounting. Also explain its Scope.
:
Definitionof Accounting :
According to American Institute of Certified Public Accountants :
Scope of Accounting :
(1) Economic Events :
(2) Identification :
(3) Measurement :
(4) Recording :
Accounting

148
footer
The number of subsidiary books to be maintained depends on the size and
nature of the business.
After recording the transactions in journal or subsidiary
books, the transactions are classified. Classification is the process of
grouping the transactions of one nature at one place, in a separate
account. The books in which various accounts are opened is called
Ledger.
Summarising involves the balancing of Ledger accounts
and the preparation of Trial Balance with the help of such Balances.
Financial Statements are prepared withthe help of trial balance. Financial
statements are includes:-
(i) Trading, Profit &Loss Account
(ii) Balance Sheet.
It refers to transmission of summarized and
interpreted informationto a variety of users. The users are:-
(i) Creditors
(ii) Investors
(iii) Lenders
(iv) Government
(v) Proprietors
(vi) Management
(vii) Banks etc.
In accounting the results of business
are presented in such a manner that the parties interested in the business
such as proprietors, managers banks, creditors etc. can have full
information about the profitability and the financial position of the
business.
Ans. Accounting is often called the language of business. The
basic function of any language is to communicate. Accounting communicates
the results of the business to the users of accounting information to enable
them to make effective decisions. To communicate information, accounting
follows a systematic process of recording, classifying and summarizing of
numerous business transactions resulting in creation of financial statements.
The two most important financial statements are:-
(iii) Trading, Profit &Loss Account.
(iv) Balance Sheet.
(5) Classification :
(6) Summarising :
(7) Communication :
(8) Interpretation of the Results :
Q. Define Accounting. Explain its Objectives Or Functions and Branches
Or Types.
:
Definitionof Accounting :
According to American Institute of Certified Public Accountants :
Accounting
149
ACCOUNTING FOR MANAGERS
footer
Accounting is the art of recording, classifying and summarizing ina significant
manner and in terms of money, transactions and events which are, in part
atleast, of a financial character, and interpreting the results thereof.
The following are the main
objectives, functions or utility of accounting:-
The main
objective of accounting is to maintain complete record of business
transactions according to some specified rules. For this purpose all the
business transactions are first of all recorded in Journal or Subsidiary
Books and thenposted into Ledger.
The second main objective of accounting
is to calculate the net profit earned or loss suffered during a particular
period. For this purpose Trading and Profit &Loss Account of the business
is prepared at the end of each accounting period. All the items relating to
purchase, sales, expenses and revenues (Income) of the business are
recorded inTrading, Profit &Loss Account.
For a businessman,
merely ascertaining profit or loss of the business is not sufficient. The
businessmanmust also knowthe financial healthof the business. For this
purpose a statement called Balance Sheet is prepared which shows the
assets onthe one hand and the liabilities and capital onthe other hand.
Another main objectives
of accounting is to communicate the accounting information to various
users.
Another objective of accounting is to
provide information about liquidity position. For this purpose it prepares
a Cash Flow Statement. It depicts inflows and outflows of cash from
operating, investing and financing activities.
One of the main objectives of accounting is to
provide bases for filing tax returns relating to income tax, sales tax, value
added tax, service tax, etc.
Branches of accounting are :
It covers the preparation and interpretation of
Objectives or Functions of Accounting:-
(1) To keep a Systematic record of business transactions :
(2) To Calculation Profit or Loss :
If Revenues >Expenses-Profit
If Revenues< Expenses-Loss
(3) To knowthe exact reasons leading to net profit or net loss.
(4) To Know the Financial Position of the business :
(5) To ascertainthe progress of the business fromyear to year.
(6) To prevent and detect errors and frauds.
(7) To Provide Informations to Various Parties :
(8) To Know the Liquidity Position :
(9) To File Tax Returns :
:
(1) Financial Accounting :
Branches ORTypes of Accounting
150
footer
financial statements and communication to the users of accounts. The
final step of financial accounting is the preparation of Trading and Profit &
Loss Account and the Balance Sheet.
The main purpose of Management
Accounting is to present the accounting information in such a way as to
assist the management in planning and controlling the operations of a
business. The management accountant uses various techniques and
concepts to make the accounting data more useful for managerial decision
making.
The branch of accounting which is used for tax
purpose is called tax accounting. Income Tax and Sale Tax are computed
onthe basis of this accounting.
The main purpose of cost accounting is to calculate
the total cost and per unit cost of goods produced and services rendered by
a business. It also estimates the cost in advance and helps the
management inexercising strict control over cost.
The society provides the
infrastructure and the facilities without which business cannot operate at
all. Hence the business also has a responsibility to the society. There is a
growing demand for reports on activities which reflect the contribution of
an enterprise to the society. Social responsibility accounting is the
process of identifying, measuring and communicating the contribution of
a business to the society. In social responsibility accounting techniques
have been developed for measuring the cost of these contribution and the
benefits to the society.
Ans. The accounting statements are needed by
various parties who have interest in the business, namely, proprietors,
investors, creditors, government and many other. Accounting statements
disclose the profitability and solvency of the business to various parties. It is
therefore, necessary that such statements should be prepared according to
some standard language and set rules. These rules are usually called General
Accepted Accounting Principles (GAAP).
Accounting principles are described by
various terms such as assumptions, conventions, concepts, doctrine,
postulate etc. These principles canbe classified mainly into two categories:-
(A) Accounting Concepts or Assumptions
(B) Accounting Conventions.
(2) Management Accounting :
(3) Tax Accounting :
(4) Cost Accounting :
(5) Social Responsibility Accounting :
Q. What do you mean by Accounting Principles or (GAAP)? Explain and
illustrate fully.
:
:
Accounting Principle
Kinds of Accounting Principles
151
ACCOUNTING FOR MANAGERS
footer
(A) Accounting Concepts or Assumptions :
(1) The Business Entity Concept :
Example :
Accounting concepts define
the assumptions on the basis of which financial statements of a business
entity are prepared. The word concept means idea or notion, which has
universal application. These accounting concepts provide a foundation for
accounting process. No enterprise can prepare its financial statements
without considering these basic concepts or assumptions. These concepts
guide how transactions should be recorded and reported. Following may
be treated as basic concepts or assumptions :
Entity concept states that business
enterprise is a separate identity apart fromits owner. Accountants should
treat a business as distinct from its owner. Business transactions are
recorded in the business books of accounts and owners transactions in
his personal books of accounts. Business unit should have a completely
separate set of books and we have to record business transactions from
firms point of viewand not fromthe point of viewof the proprietor.
Kinds of Accounting Principles
Accounting
Conventions
Convention of Full Disclosure
Convention of Consistency
Convention of Conservatism
Convention of Materiality
Business Entity Concept
Money Measurement Concept
Going Concern Concept
Accounting Period Concept
Historical Cost Concept
Dual Aspect Concept
Revenue Recognition Concept
Matching Concept
Accrual Concept
Objectivity Concept
Accounting Concepts
or Assumptions
152
footer
(i) The proprietor is treated as a creditor of the business to the extent of
capital invested by him in the business. The capital is treated as a
liability of the firm because it is assumed that the firm has borrowed
funds from its own proprietors instead of borrowing from outside
parties. It is for the reason that we also allow interest on capital and
treat it as anexpense of the business.
(ii) Similarly, the amount withdrawn by the proprietor fromthe business
for his personal use is treated as his drawings.
(iii) The proprietors house, his personal investment in securities, his
personal car and personal income and expenditure are kept separate
fromthe accounts of the business entity.
(iv) If the proprietor has some other business entity doing another
business, the records of that business should also be kept separate.
The concept of separate entity is applicable to all forms of business
organizations, i.e. sole proprietorship, partnership or a company.
As per this concept, only those
transactions, which can be measured in terms of money are recorded.
Transactions, even if, they affect the results of the business materially, are
not recorded if they are not convertible in monetary terms. Transactions
and events that cannot be expressed in terms of money are not recorded in
the business books. For example, accounting does not record a quarrel
between the production manager and sales manager; it does not report
that a strike is beginning and it does not reveal that a competitor has
placed a better product in the market. These facts or happenings cannot
be expressed inmoney terms and thus are not recorded inthe books.
A business on a particular day has 5000 Kilograms of raw
materials, 5 Machines, 100 Chairs and 20 Fans. All these things cannot be
added up unless expressed in terms of money. In order to make a record of
these items, these will have to be expressed in monetary terms such as Raw
Materials Rs. 25000, Machines Rs. 200000, Chairs Rs. 5000 and Fans Rs.
8000. As such, to make accounting records relevant, simple, understandable
and homogeneous, they are expressed in a common unit of measurement i.e.,
money.
As per this concept it is assumed that the
business will continue to exist for a long period in the future. The
transactions are recorded in the books of the business on the assumption
that it is a continuing enterprise.
(i) It is on this concept that we record fixed assets at their original cost
and depreciation is charged on these assets without reference to their
market value.
(2) Money Measurement Concept :
Example :
(3) Going Concern Concept :
Example :
153
ACCOUNTING FOR MANAGERS
footer
(ii) It is also because of the going concern concept that outside parties
enter into long-term contracts with the enterprise, gives loans and
purchase the debentures and shares of the enterprise.
(iii) Another example of this concept is that Prepaid Expenses, which
have no realizable value are shown as assets in the balance sheet,
because the benefits of suchexpenses will be received infuture.
According to this concept accounts
should be prepared after every period & not at the end of the life of the
entity. Usually this period is one calendar year i.e. 1 Jan to 31 December
or from 1 April to 31 March. According to Amended Income Tax Law, a
business has compulsorily to adopt financial year beginning on 1 April
and ending on 31 March. Apart from this, companies whose shares are
listed on the stock exchange are required to publish quarterly results to
depict the profitability and financial position at the end of three months
period.
According to this concept,
an asset is ordinarily recorded in the books of accounts at the price at
which it was purchased or acquired. This cost becomes the basis of all
subsequent accounting for the asset. Since the acquisition cost relates to
the past, it is referred to as historical cost. This cost is the basis of
valuationof the assets inthe financial statements.
If a business purchases a building for Rs. 500000, it would be
recorded in the books at this figure. Subsequent increase or decrease in
the market value of the building would not be recorded in the books of
accounts.
(i) It is highly objective and free frombias.
(ii) Market values of assets are difficult to be determined.
(iii) Market values of the assets may change from time to time and it will
be extremely difficult to keep track of up and down of the market
price.
(i) Assets for which nothing is paid will not be recorded. Thus a
favourable location, brand name and reputation of the business,
knowledge and technological skill built inside the enterprise will
remainunrecorded thoughthese are valuable assets.
(ii) Historical cost-based accounts may lose comparability.
(iii) Many assets do not have acquisitioncost.
(iv) During periods of inflation, the figure of net profit disclosed by profit
and loss account will be seriously distorted because depreciation
(4) Accounting Period Concept :
(5) Historical Cost Concept or Cost Concept :
Example :
Benefits :
Limitations :
st st
st st
st
st
154
footer
based on historical costs will be charged against revenues at current
prices.
(v) Information based upon historical cost may not be useful to
management, investors, creditors etc.
According to this concept, every business
transaction is recorded as having a dual aspect. In other words, every
transaction affects atleast two accounts. If one account is debited, any
other account must be credited. The system of recording transactions
based on this concept is called as Double Entry System. It is because of
this principle that two sides of the Balance Sheet are always are equal and
the following accounting equation will always hold good at any point of
time:-
X commences business with Rs. 5 Lacs in cash and takes a
loan of Rs. 1 Lac fromthe bank, and these 6 Lacs are used in buying some
assets, say, plant &machinery. The equationwill be as follows:
Assets = Liabilities + Capital
Rs. 6 Lacs = Rs. 1 Lac + Rs. 5 Lacs
Revenue means the
amount which is added to the capital as a result of business operations.
Revenue is earned by sale of goods or by providing a service. Concept of
revenue recognition determines the time or the particular period in which
the revenue is realized. Revenue is deemed to be realized when the title or
ownership of the goods has been transferred to the purchaser and when
he has legally become liable to pay the amount. It should be remembered
that revenue recognitionis not related withthe receipt of cash.
For example, if a firm gets an order of goods on 1 January,
supplies the goods on 20 January and receives the cash on 1 April, the
revenue will be deemed to have been earned on 20 January, as the
ownership of goods was transferred onthat day.
This concept is very important for correct
determination of net profit. According to this concept, all expense are
matched with the revenue of that period should only be taken into
consideration. This principle is based on accrual concept as it considers
the occurrence of expenses and income and do not concentrate on actual
inflow or outflow of cash. This principle helps us in finding Net profit or
Loss. Following points must be considered while matching costs with
revenue:
(6) Dual Aspect Concept :
Assets = Liabilities + Capital
OR
Capital = Assets - Liabilities
Example :
(7) Revenue Recognition (Realisation) Concept :
Example :
(8) Matching Concept :
st
th st
th
155
ACCOUNTING FOR MANAGERS
footer
(i) When an itemof revenue is included in the profit and loss account, all
expenses incurred on it, whether paid or not, should be show as
expenses inthe profit and loss account.
(ii) When some expenses, say insurance premium is paid partly for the
next year also, the part relating to next year will be shown as an
expense only next year and no this year.
(iii) Similarly, income receivable must be added in revenues and incomes
received inadvance must be deducted fromrevenues.
In accounting, accrual basis is used for recording
transactions. It provides more appropriate information about the
performance of business enterprise as compared to cash basis. Accrual
concept applies equally to revenues and expenses. In accrual concept
revenue is recorded when sales are made whether cash is received or not.
Similarly, according to this concept, expenses are recorded in the
accounting period in which they assist in earning the revenues whether
the cashis paid for themor not.
This concept requires that accounting
transaction should be recorded in an objective manner, free from the
personal bias of either management or the accountant who prepares the
accounts.
An accounting convention may be defined as
a customor generally accepted practice which is adopted either by general
agreement or common consent among accountants. Accounting
conventions differ fromconcept inrespect to the following:
(i) Accounting concepts are established by law while accounting
conventions are guidelines based upongeneral agreement.
(ii) There is no role of personal judgment or individual bias in the
adoption of accounting concepts whereas they may play a crucial role
infollowing accounting conventions.
(iii) There is uniform adoption of accounting concepts in different
enterprise while it may not be so incase of accounting conventions.
This principle requires that all
significant information relating to the economic affairs of the enterprise
should be completely disclosed. The principle is so important that the
companies Act makes ample provisions for the disclosure of essential
information in the financial statements of a company. The proforma and
contents of Balance Sheet and Profit and Loss Account are prescribed by
Companies Act. Various items or facts which do not find place in
accounting statements are shown in the Balance Sheet by way of
footnotes. Suchas :
(9) Accrual Concept :
(10) Objectivity Concept :
(B) Accounting Conventions :
Following are the mainaccounting conventions :
(1) Conventions of Full Disclosure :
156
footer
(i) Contingent Liabilities.
(ii) If there is a change in the method of valuation of stock, or for
providing depreciation or in making provision for doubtful debts, it
should be disclosed inthe Balance Sheet by way of a footnote.
(iii) Market value of investments should be givenby way of a footnote.
According to this principle, accounting
principles and methods should remain consistent from one year to
another. These should not be changed from year to year. If a firm adopts
different accounting principles in two accounting periods, the profits of
current period will not be comparable with the profits of the preceding
period.
According to this principle, all
anticipated losses should be recorded in the books of accounts, but all
anticipated gains should be ignored. In other words, conservatism is the
policy of playing safe. When there are many alternative values of an asset,
anaccountant should choose the method whichleads to the lesser value.
(i) Valuationof closing stock cost or market price whichever is less.
(ii) Provisionfor Doubtful Debts onDebtors.
(iii) Joint Life Policies are recorded at Surrender Values.
Effects of Principle of Conservatism:
(i) Profit & Loss account will disclose lower profits in comparison to the
actual profits.
(ii) Balance sheet will discloses understatement of assets and
overstatement of liabilities incomparisonto the actual values.
This convention is an exception to the
convention of full disclosure. According to this convention, all the items
having significant economic effect should be disclosed in financial
statements and any insignificant itemwhich will only increase the work of
the accountant should not be disclosed in the financial statements. It
should be noted that what is material for one concern may be immaterial
for another. Thus, the accountant should judge the important of each
transactionto determine its materiality.
Ans. Classificationof Accounts are:
(2) Convention of Consistency :
(3) Convention of Conservatism :
Examples of the application of the principle of conservatism:
(4) Convention of Materiality :
Q. Give ClassificationOf Accounts. What are the Rules of Journalising?
: Classificationof Accounts
Classification of Accounts
Personal Accounts Impersonal Accounts
Real Accounts Nominal Accounts
157
ACCOUNTING FOR MANAGERS
footer
1. Personal Accounts :
Natural Personal Accounts :
Artificial Personal Account :
Representative Personal Accounts :
Golden Rule of Personal Account :
2. Impersonal Account :
Real Account :
Golden Rule of Real Account :
Nominal Account :
Golden Rule of Nominal Account :
Q. Define Accounting Cycle ORProcess of Accounting.
:
The accounts which relate to an individual, firm,
company or an institution are called personal accounts. Account of
Mohan, Account of D.C.M. Limited, Capital Account of proprietor, etc. are
the examples of Personal Accounts. This account is further classified into
three categories:-
(i) It relates to transactions of human
beings like Ram, Rita, etc.
(ii) These accounts do not have a
physical existence as human beings but they work as personal
accounts. For example: Government, Companies (private or limited),
Clubs, Co-operative Societies etc.
(iii) These are not in the name of
any person or organization but are represented as personal account.
For Example: Outstanding liability account or prepaid account,
capital account, drawings account.
Debit the Receiver
Credit the Giver
Accounts which are not personal such as
machinery account, cash account, rent account etc. These can be further
sub-divided as follows :
(i) Accounts which relate to assets of the firm but not
debt. For example accounts regarding Land, Building, Investment,
Fixed Deposits etc., are real accounts Cash-in-hand and Cash at
Bank are also real.
Debit what comes in.
Credit what goes out.
(ii) Accounts which relates to expenses, losses,
gains, revenue etc. like salary account, interest paid account,
commissionreceived account.
Debit all expense &Losses.
Credit all Incomes &Gains.
Ans. An accounting cycle is a complete sequence of
accounting procedures which are repeated in the same order during each
accounting period. The accounting cycle may be shownas below:-
Accounting Cycle
158
footer
(1) Identification of Transaction :
(2) Journal :
PROFORMA OF JOURNAL
Date :
Particulars :
Accounting deals with business
transactions which are monetary in nature. In other words, the
transactions which cannot be measured and expressed in terms of money
cannot be recorded inaccounting.
Journal is one of the basic book of original entry in which
transactions are recorded in a chronological (day-to-day) order according
to the principles of double entry system. When the size of business is a
small one, it may be possible to record all transactions in the journal but
whenthe size of the business grows and the number of transactions is very
large journal is sub-divided into a number of books called subsidiary
Books.
There are five columns injournal whichare:-
(i) In the first column, date of transaction is entered. The year
and monthis writtenonly once, till they change.
(ii) Each transactions affects two accounts out of which
one account is debited and other account is credited.
Books of Original Entry:
1. Cash Book
2. Purchase Book
3. Sales Book
4. Purchase Return Book
5. Bills Receivable Book
6. Bills Payable Book
7. Journal Proper
Journal
Ledger
Trial Balance
Trading, Profit &
Loss A/c and
Balance Sheet
Transactions
Diagram : Accounting Cycle
Date Particulars L.F. Amount Dr. Amount Cr.
(1) (2) (3) (4) (5)
159
ACCOUNTING FOR MANAGERS
footer
(iii) All entries from the journal are later posted
into the ledger accounts. The page number of the ledger account
where the posting has been made from the journal is recorded in the
L.F. columnof the journal.
(iv) Inthe fourthcolumn, the amount of the account being
debited is written.
(v) In the fifth column, the amount of the account being
credited is written.
Business transactions are first recorded in journal or
Subsidiary books. The next step is to transfer the entries to respective
accounts inledger. This process is called ledger
Each ledger account is divided into two equal parts. The left-hand side is
knownas the debit side and the right-hand side as the credit side.
As shownabove, there are four columns oneachside of anaccount:-
(i) The date of the transactionis recorded inthis column.
(ii) Eachtransactionaffects two accounts.
(iii) In this column page number of the journal or
subsidiary book from which the particular entry is transferred, is
entered.
(iv) The amount is entered inthis column.
When posting of all the transactions into ledger is
completed and the accounts are balanced off, it becomes necessary to
check the arithmetical accuracy of the accounting work. For this purpose,
the balance of each and every account in the ledger is put on a list. The list
so prepared is called a trial balance.
(i) It is a list of balances of all ledger accounts and the cashbook
Ledger Folio or L.F. :
Amount Dr. :
Amount Cr. :
(3) Ledger :
Date :
Particulars :
Journal Folio or J.F. :
Amount :
(4) Trial Balance :
PROFORMA OF TRIAL BALANCE
Features of a Trial Balance :
Date Particulars J.F. Amount Date Particulars J.F. Amount
Dr. Cr.
Name of the Accounts L.F. Dr. Balances Cr. Balances
160
footer
(ii) It is just a statement and not anaccount.
(iii) It is neither a part of double entry system, nor does it appear in the
actual books of accounts. It is just a working paper.
(iv) It can be prepared at any time during the accounting period, say, at
the end of every month, every quarter, every half year or every year.
(v) It is always prepared on a particular date and not for a particular
period.
(vi) It is prepared to check the arithmetical accuracy of the ledger
accounts.
(vii) If the books are arithmetically accurate, the total of all debit balances
of a trial balance will be equal to the total of all credit balances.
(i) To ascertainthe arithmetical accuracy of the ledger accounts.
(ii) To help inlocating errors
(iii) To obtaina summary of the ledger accounts
(iv) To help inthe preparationof final accounts.
After having
checked the accuracy of the book of accounts through preparation of Trial
Balance, businessman wants to ascertain the profit earned or loss
suffered during the year and also the financial position of his business at
the end of the year. For this purpose he prepares Final Accounts which
are also termed as Financial Statements. These include the following:-
Trading Account.
Profit and Loss Account.
Balance Sheet.
Ans. According to Double Entry System, every
transaction has two fold-aspects- debit and credit and both the aspects are to
be recorded in the books of accounts. We may define the Double Entry System
as the system which records both the aspects of transactions. This principle
proves accounting equationi.e. bothsides of Balance Sheet always equal.
Assets = Liabilities + Capital
This system affords the under
mentioned advantages :
(1) Scientific System
(2) Complete Record of Every Transaction
(3) Preparationof Trial Balance
(4) Preparationof Trading &Profit &Loss A/c
(5) Knowledge of financial positionof the Business
(6) Knowledge of Various Informations.
Objectives of Preparing Trial Balance :
(5) :
Q. Write a Short Note OnDouble Entry System.
:
Advantages of Double Entry System :
Trading, Profit & Loss Account And Balance Sheet
Double Entry System

161
ACCOUNTING FOR MANAGERS
footer
(7) Comparative Study
(8) Lesser possibility of Fraud.
(9) Help management indecisionmaking.
(10) Legal Approval
(11) Suitable for All types of Businessmen.
Ans. In every business there are certain assets of a fixed
nature that are needed for the conduct of business operations. Some examples
of such assets are Building, Plant & Machinery, Motor Viechles, Furniture,
office Equipments etc. These assets have a definite span of life after the expiry
of which the assets will lose their usefulness for the business operations. Fall in
the value &utility of such assets due to their constant use and expiry of time is
termed as depreciation.
Depreciation may be defined as the permanent and continuing
diminutioninthe quality, quantity or the vale of anasset.
1. Depreciationis decline inthe value of fixed assets (except Land)
2. Suchfall is of a permanent nature.
3. Depreciation is a continuous process because value of assets will
decline by their constant use.
4. Depreciation decreases only the book value of the asset, not the
market value.
5. Depreciation is a non-cash expense. It does not involve any cash
outflow.
1. By Constant Use.
2. By Obsolescence
3. By expiry of time.
4. By Accident.
5. By expiry of legal rights.
6. By Depletion
7. By permanent fall inmarket price.
1. For ascertaining the truthprofit or loss.
2. For showing the truthtrue and fair view of the financial position.
3. To ascertainthe accurate cost of production.
Q. Define Depreciation. What are the Causes & Methods of
Depreciation?
:
:
According to WilliamPickles
:
:
:
Depreciation
Definitionof Depreciation
Features of Depreciation
Causes of Depreciation
Need, Importance or objects of providing depreciation
162
footer
4. To provide funds for replacement of assets.
5. To prevent the distributionof profits out of capital.
6. For avoiding over payment of Income tax.
7. Other objectives.
1. Total Cost of the Asset.
2. Estimated life of Asset.
3. Estimated Scrap Value.
1. Straight Line Method.
2. WrittenDownValue Method.
3. Annuity Method.
4. DepreciationFund Method.
5. Insurance Policy Method.
6. RevaluationMethod.
7. DepletionMethod.
8. Machine hour rate Method.
Ans. This method is also termed as Original Cost
Method because under this method depreciation is charged at a fixed
percentage on the original cost of the asset. The amount of depreciation
remains equal from year to year and as such this method is also known as
Equal Installment Method, or Fixed Installment Method. Under this method,
the amount of depreciation is calculated by deducting the scrap value fromthe
original cost of the asset and then by dividing the remaining balance by the
number of years of its estimated life.
Original Cost of the Asset Estimated Scrap Value

Estimated Life of the Asset.


1. Calculation of Depreciation under this method is very
simple and as suchthe method is widely popular.
2. Under this method, equal amount of
depreciation is debited to profit & loss account of each year. Hence, the
burdenof depreciationoneachyears net profit is equal.
3. Under this method, the book
value of an asset can be reduced to net scrap value or zero value, which is
not possible under some other methods.
Factors determining the amount of Depreciation
Methods of providing or Allocating Depreciation
Straight Line Method
:
:
Q. Explain Straight Line Method of Depreciation with the help of an
Example.
:
Yearly Depreciation=
Merits of Straight Line Method :
Simplicity :
Equality of Depreciation Burden :
Assets can be completely written off :
163
ACCOUNTING FOR MANAGERS
footer
4. under this
method, the original cost of the asset is shown in the Balance Sheet and
the upto-date depreciationis shownas a direct deductionfromit.
1. When there are different machines having
different life-span, the computation of depreciation becomes complicated
because depreciation on each machine will have to be calculated
separately.
2. Repairs charges go on increasing year
by year as the asset becomes older but as the equal depreciation is
charged under this method eachyear.
3. This method does not take into
considerationthe loss of interest onthe amount invested inthe asset.
4. Sometimes, even after
the value of anasset is reduced to zero inthe books, it continues to be used
inthe business inactual practice
5. It is quite difficult to
assess the true scrap value of the asset after a long period, say 15 or 20
years fromthe date of its installation.
This method is suitable for those assets whose useful life can be
renewals.
Birla Cotton Mills purchased a machinery on 1 May, 1991 for Rs. 90,000. On 1
July, 1992 it purchased another machinery for Rs. 40,000.
On 31 March, 1993 it sold off the first machine purchased on 1991 for Rs.
58,000 and on the same date purchased a new machinery for Rs. 1,00,000.
Depreciation is provided at 20%p.a. on the original cost method. Accounts are
closed eachyear on31 December. Showthe Machinery Account for three years
Knowledge of original cost and upto date depreciation :
:
Difficulty in Computation :
Unequal pressure in later years :
Omission of Interest factor :
Unrealistic to write off the vale of asset to zero :
Difficulty in the determination of scrap value :
:
:
Dr. Machinery Account Cr.
Demerits of Straight Line Method
Suitability
Example
st st
st
st
Date Particulats J.F. Amount Date Particulars J.F. Amount
1991 1991
May 1 To Bank A/c 90,000 Dec.31 By depreciationA/c 12,000
(for 8 months)
Dec.31 By Balance C/d 78,000
90,000 90,000
164
footer
Q. Discuss the Merits And Demerits Of Providing Depreciation By
Diminishing Balance Method?
: Ans. Under this method, as the value of asset
goes on diminishing year after year, the amount of depreciation charged every
year also goes ondeclining.
Written Down Value Method
1992 1992
Jan1 To Balance B/d 78,000 Dec31 By DepreciationA/c
July1 To Bank A/c 40,000 (i) 18,000
(ii) 4,000 22,000
(for 6 months)
Dec31 By Balance C/d
(i) 60,000
(ii) 36,000 96,000
1,18,000 1,18,000
1993 1993
Jan1 To Balance B/d Mar.31 By Bank A/c 58,000
(i) 60,000 Mar.31 By Dep. A/c 4,500
(ii) 36,000 96,000 (for 3 months)
Mar. To Bank A/c 1,00,000 Dec. 31 By Dep. A/c
31 (ii) 8,000 23,000
Mar. To Profit &Loos (iii) 15,000
31 A/c (Profit On
machine)
Rs. 58,000+
4,500-60,000 2,500 Dec.31 By Bal. C/d
(ii) 28,000
(iii) 85,000 1,13,000
1,98,500 1,98.500
1994
Jan.1 To Bal. B/d 1,13,000
(ii) 28,000
(iii) 85,000
165
ACCOUNTING FOR MANAGERS
footer
For Example
:
Easy Calculation :
Equal Charge against income :
No induce pressure in later years :
Balance of asset is never written off to zero :
Approved method by Income Tax Authorities :
:
Asset can not be completely written off. :
Omission of Interest Factor :
Difficulty in determining the rate of depreciation :
Knowledge of original cost & up to date depreciation not possible :
:
if a machine is purchased for Rs. 10,000 and depreciation is to be
charged at 10% p.a. according to written down value method, the depreciation
will be charged as under:-
1 Year onRs. 10,000 @10% =1,000
2 Year onRs. 9,000 (10,000-1,000) @10% = 900
3 Year onRs. 8,100 (9,000-900) @10% = 810
and so on.
It will be observed from the above calculations that each years depreciation is
calculated on the book value of the asset at the beginning of that year, rather
than on the original cost. As the value of asset and also the depreciation
charged on its goes on reducing year after year, this method is known as
Reducing Installment Method.
1. It is easy to calculate the depreciation under this
method, evenif some newassets are purchased year after year.
2. In this method, the total burden on
profit & Loss account in respect of depreciation and repairs put together
remains almost equal year after year.
3. The efficiency of machine is more in
the earlier years than in later years. Hence, the depreciation in first few
years should be more incomparisonto the later years.
4. This method ensures that
the assets is never reduced to zero.
5. This method of
providing depreciationis permissible under Income Tax Regulations.
1. Under this method, the value
of an asset, even if it becomes obsolete and useless, cannot be reduced to
zero and some balance, however small, would continue onasset account.
2. This method does not take into
considerationthe loss of interest onthe amount invested inthe asset.
3. Under this
method, the rate of providing depreciationcannot be easily decided.
4.
Under this method, the original cost of various assets is not shown in the
Balance Sheet.
A company had bought machinery for Rs. 100000 including there
st
nd
rd
Merits of WrittenDownValue Method
Demerits of WrittenDownValue Method
Example
166
footer
in a boiler worth Rs 10000 depreciation was charged on reducing balance
method at the rate of 10% p.a. for first five year and machinery account was
credited accordingly. During the fifth year, the boiler becomes useless on
account of damages. The damaged boiler is sold for Rs. 2000 prepares the
machinery account for five years.
MACHINERY ACCOUNT
Date Particulars Amount Date Particulars Amount
Year To Bank A/c 90000 Year By Dep.
Ist To Bank A/c 10000 Ist (i) 9000 10000
(Boiler) (ii) 1000
By Bal. C/d
(i) 81000
(ii) 9000 9000
100000 100000
Year To Bal. B/d Year By Dep.
II (i) 81000 II (i) 8100
(ii) 9000 (ii) 900 9000
90000 By Bal. C/d
(i) 72900 81000
(ii) 8100
90000 90000
Year To Bal. B/d Year By Dep.
III (i) 72900 III (i) 7290
(ii) 8100 (ii) 810
81000 By Bal. C/d 8100
(i) 65,610
(ii) 7290
81000 81000
Year To Bal. B/d Year By Dep.
IV (i) 65610 72900 IV (i) 6561 7290
(ii) 7290 (ii) 729
By Bal. C/d
(i) 59049 65610
(ii) 6561
72900 72900
Year To Bal. B/d Year By Bank 2000
V (i) 59049 V By P & L A/c 4561
(ii) 6561 65610 (6561-2000)
By Dep. 5905
By Bal. C/d 53144
65610 65610
Year To bal B/d 53144
VI
Dr. Cr.
167
ACCOUNTING FOR MANAGERS
footer
Q. What do you mean by Final Accounts? What is their Necessity?
:
:
:
Format of a Trading Account: Trading Account
(for the year ending)
Dr. Cr.
Ans. Financial Statements refers to such statements which
report the profitability and the financial position of the business at the end of
accounting period. The termfinancial statements include the following:-
(1) Trading Account
(2) Profit and Loss Account.
(3) Balance Sheet
(1) Trading account is prepared for calculating the gross
profit or gross loss arising or incurred as a result of the trading activities of
a business. In other words, it is prepared to show the result of
manufacturing, buying and selling of goods.
(i) It provides informationabout Gross Profit and Gross Loss.
(ii) It provides informationabout the direct expenses.
(iii) Comparisonof closing stock withthose of the previous years.
(iv) It provides safety against possible losses.
Final Accounts
Trading Account
Need and Importance of Trading Account
Particulars` Amount Particulars Amount
Rs. Rs.
To Opening Stock By Sales
To Purchases Loss Sales Return
Less : Purchase Reture OR
OR Returns In wards
By Closing Stock
To Carriage on Purchase
To Gas, Fuel and Power
To Freight, Octroi and Cartage
To Manufacturing Expenses
or Productive Expenses.
To Factory Expense, Such as
Factory Lighting, Factor Rent Etc.
To Dock Charges
To Import duty or Custom Duty
To Royalty
To Gross Profit
Transferred to P & L A/c
(Balancing Figure)
Return Outward
To Wages
To Wages & Salaries By Gross Loss (if any)
To Direct Expenses Transferred to P & L A/c
To Carriage or (Balancing Figure)
To Carriage Inwards or
168
footer
(2) Trading account only disclose the gross profit
earned as a result of buying and selling of goods. However, a businessman
has to incurr a number of expenses which are not taken into trading
account. Hence a businessman is more interested in knowing the net
profit earned or net loss incurred during the year.
A profit and loss account is an account into which all gains and losses are
collected, in order to ascertain the excess of gains over the losses or vice-
versa.
(i) To Ascertainthe Net Profit &Net Loss
(ii) Comparisonwithprevious years profit.
(iii) Control onExpenses
(iv) Helpful inpreparationof the balance Sheet
Profit & Loss Account
Need and Importance of Profit &Loss Account
:
:
Format of Profit And Loss Account : Profit And Loss A/c
( for the year ending __________________)
Particulars` Amount Particulars Amount
Rs. Rs.
To Gross Profit B/d By Gross Prfit B/d
(transferred from trading A/c) (Transferred from trading
A/c)
To Salaries By Rent form tenant
To Salaries & Wages By Discount Received
To Rent, Rate and Taxes By Commission Received
To Printing & Staionery By Any Other Income
To Lighting By Net Loss (if any)
To Telephone Charges Transferred to Capital A/c
To Audit Fees etc.
To Carriage outward or Carriage
on sales
To Advertisement
To Commission
To Bed-Debts
To Export Duty
To Parcking Exp etc.
To Discount/Discount Allowed
To Repairs
To Depreciation
To Interest
To Bank Charges etc.
To Net Profit
(Transferred to Capital A/c)
To Office Expenses :
To Selling & Distribution Expense:
To Miscellaneous Expenses :
169
ACCOUNTING FOR MANAGERS
footer
(3) After ascertaining the net profit or net loss of the
business enterprise, the businessman would also like to know the exact
financial position of his business. For this purpose a statement is
prepared which contains all the assets and liabilities of the business
enterprise. The statement so prepared is called a Balance Sheet.
Balance Sheet :
Balance Sheet
(As on Or As At --------------)
Particulars` Amount Particulars Amount
Rs. Rs.
Bank Overdraft Cash-in-Hand
Bill Payable Cash at Bank
Sundry Creditors Bills Receivables
Outstanding Expenses Short Term Investments
Unearned Income Sundry Debtors
Closing Stock
Prepaid Expenses
Long Term Loans Accrued Income
Furniture
Loose Tools
Add: Net profit Motor Vehicle
Less: Drawings Long-term investments
Less: Income Tax Plant & machinery
Less: Life Insurance Premium Land & Building
Less: Net Loss Patents
Goodwill
Current Liabilities : Current Assets :
Fixed Liabilities :
Reserves: Fixed Assets:
Capital:
Need and Importance of Balance Sheet :
Q. What is the necessity of doing adjustments? Give some adjustment
entries withtheir explanation.
:
1. The main purpose of preparing balance sheet is to ascertain the true
financial positionof the business at a particular point of time.
2. It gives exact information about the exact amount of capital at the end of
the year and the additionor deductionmade into it inthe current year
3. It helps infinding out whether the firmis solvent or not.
4. It helps inpreparing the opening entries at the beginning of the next year.
Ans. In order to ascertain the true profit or loss of the business
for a particular year, it is necessary that all expenses and incomes relating to
that year are taken into consideration. For example, if we want to ascertain the
net profit for the year ended on 31 December and rent for the month of
Adjustments
st
170
footer
December has not yet been paid, it would be proper to include such rent along
with the other expenses of the year. Similarly, it often happens that certain
incomes, like interest, dividend, etc. are earned but not received during the
year. Adjustment for such incomes must be made in the current year itself, so
that the profit and loss account may disclose the correct amount of net profit or
loss and the balance sheet may present the true financial position of the
business.
Simply stated, while preparing final accounts it must be detected whether there
is a transaction
(i) Whichhas beenomitted to be recorded inthe books, or
(ii) Whichhas beenwrongly recorded inthe books, or
(iii) Of whichonly one aspect has beenrecorded inthe books.
Entries passed for suchtransactions are called adjustment entries.
(1) To ascertainthe true Net Profit or loss of the business.
(2) To ascertainthe true financial positionof the business.
(3) To make a record of the transactions omitted fromthe books
(4) To rectify the errors committed inthe books of accounts
(5) To make a record of such expenses which have been accrued but
have not beenpaid.
(6) To make a record of such incomes which have accrued but have not
beenreceived.
(7) To provide for depreciationand other provisions.
(1) The amount of goods unsold at the end of the year is
called closing stock. It is valued at Cost Price or Realisable Value,
whichever is less. The basic principle underlying the valuation of closing
stock is that anticipated losses should be taken into account, but all
unrealized gains should be ignored.
(i) If the closing stock appears outside the Trial Balance, it will be shown
at two places, i.e., on the Credit side of the Trading A/c and on the
Assets side of the Balance Sheet.
(ii) If the closing stock appears inside the Trial Balance, it will be shown
only onthe Assets side of the Balance Sheet.
(2) These are the
expenses which have been incurred during the year but have been unpaid
onthe date of preparationof final accounts.
(i) If outstanding expenses have been mentioned inside the Trial
Need of Adjustments
Explanation of Important Adjustments
:
:
Closing Stock :
Treatment inFinal Accounts :
Outstanding Expenses Or Expenses Due but not Paid :
Treatment inFinal Accounts :
171
ACCOUNTING FOR MANAGERS
footer
Balance, they will be shownonthe liabilities side only.
(ii) If outstanding expenses have been mentioned outside the Trial
Balance, then on the one hand, it will be added to the concerned
expenses on the debit side of Trading or Profit and Loss Account and
on the other hand, will also be shown on the liabilities side of the
Balance Sheet.
(3)
These are the expenses which have been paid in advance for the
next year during the current year itself.
Treatment inFinal Accounts :
(i) If Prepaid expenses have been mentioned inside the Trial Balance,
they will be shownonthe Assets side only.
(ii) If Prepaid expenses have been mentioned outside the Trial Balance,
then on the one hand, it will be deducted from the concerned
expenses on the debit side of Trading or Profit and Loss Account and
on the other hand, will also be shown on the Assets side of the
Balance Sheet.
(4) Depreciation is the loss or fall in the value of fixed assets
due to their constant use and expiry of time.
Depreciation on the one hand, will be
shown on the debit side of the Profit and Loss Account and on the other
hand, will also be deducted from the value of the concerned asset on the
Asset side of the Balance Sheet.
(5) It is quite commonthat certain
items of income such as interest, commission etc are earned during the
current year but have not been actually received by the end of the current
year. Suchincomes are knownas Accrued Incomes or Earned Incomes
(i) If accrued incomes have been mentioned inside the Trial Balance,
they will be shownonthe Assets side only.
(ii) If Accrued incomes have been mentioned outside the Trial Balance,
then on the one hand, It will be shown on the credit side of the Profit &
Loss Account and on the other hand, will be shown on the assets side
of the Balance Sheet.
(6) It may also
happen that a certain income is received in the current year but the whole
amount of it does not belong to the current year. Such portion of this
income which belongs to the next year is known as Unearned Income or
Income received but not earned.
Prepaid expenses Or Unexpired Expenses Or Expenses Paid in
Advance :
Depreciation :
Treatment in Final Accounts :
Accrued Income or Income Receivable :
Treatment inFinal Accounts :
Unearned Income Or Income Received in Advance :
172
footer
(i) If Unearned incomes have been mentioned inside the Trial Balance,
they will be shownonthe Liabilities side only.
(ii) If Accrued incomes have been mentioned outside the Trial Balance,
then on the one hand, It will be deducted fromthe concerned income
on the Credit side of the Profit &Loss Account and on the other hand,
will be shownonthe Liabilities side of the Balance Sheet.
(7) Usually in order to ascertain the true efficiency of
the business, interest at a normal rate is charged on the capital invested
by the proprietor inthe business.
Interest on capital is an expense for the
business and hence it is shown on the debit side of Profit &Loss Account.
At the same time, it is a gain to the proprietor and hence is added to his
capital.
(8) Occasionally, the proprietor draws cash or goods
for his personal use. Such withdrawals are terms as Drawings. If the firm
pays interest on capital, it is fully justified that it should also charge
interest ondrawings.
Interest on drawings is a gain to the
business and hence it is shown on the credit side of Profit &Loss Account.
At the same time, it is an expense fromthe proprietors viewand hence will
be deducted fromthe capital.
(9)
(i) Generally, item of Loan appears on the credit side of the Trial
Balance. It means that the amount has been borrowed from some
person or the bank etc. Loan is a liability of the firm and the interest
on such loan will be an expense. It up-to-date interest has not been
paid on the Loan, the unpaid interest will have to be calculated and
will be treated just like outstanding expenses.
Treatment in Final Accounts : When Loan appears on the credit side
of the Trial Balance, interest on it will be an expense and hence will be
recorded on the debit side of Profit & Loss Account. Outstanding
amount of such interest will also be added to Loan Account on the
Liabilities side of the Balance Sheet.
(ii) On the contrary, if the item of loan appears on the debit side of Trial
Balance, it will mean that the amount has been lent to outsider. It will
be an asset in this case and interest on such loan will be an income
for the firm.
When Loan appears on the Debit
side of the Trial Balance, interest on it will be an income and hence
will be recorded on the credit side of Profit & Loss Account and will
also be added to Loan Account on the assets side of the Balance
Sheet.
Treatment inFinal Accounts :
Interest on Capital :
Treatment in Final Accounts :
Interest on Drawings :
Treatment in Final Accounts :
Interest on Loan :
Treatment in Final Accounts :
173
ACCOUNTING FOR MANAGERS
footer
(10) Persons to whom goods have been sold on credit are known
as Debtors. Sometimes due to the dishonesty, death or insolvency of a
debtor, full amount is not received from him. When it becomes certain
that a particular amount will not be recovered it is known a s B a d -
Debts.
(i) If Bad-debts are given in the adjustments or outside the Trial
Balance, they will be shown on the debit side of the Profit & Loss
Account and will also be deducted fromthe Debtors onthe assets side
of the Balance Sheet.
(ii) If Bad-Debts are given inside the trial balance, it will be shown on the
debit side of the Profit &Loss Account.
(11) Even after deducting the
amount of actual bad-debts fromthe debtors, the list of debtors at the end
of the year include some debts which are either bad or doubtful. A
provision is created to cover any possible loss on account of bad-debts
likely to occur in future. Such a provision is created at a fixed percentage
ondebtors every year and is called provisions for bad and doubtful debts.
Treatment in Final Accounts : The amount of provision for doubtful debts
onthe one hand, is shown on the debit side of the Profit and Loss
Account and onthe other hand, is deducted from Sundry debtors on the
assets side of the Balance Sheet.
(12) It is a normal practice in the
business to allow cash discount to those debtors fromwhomthe payment
is received promptly or with a fixed period. Discount thus allowed will be
an expense of the business. It should be noted that discount will be
allowed only to those debtors who will make prompt payment.
Such provision is shown on the debit
side of the profit & loss account and is also deducted from Sundry
Debtors onthe Assets side of the Balance Sheet.
(13) Such provision is shown on the
credit side of the Profit & Loss account and is also deducted from the
Sundry Creditors onthe Liabilities side of the Balance Sheet.
(14) Sometimes losses occur due to some abnormal
circumstances such as accident, fire, flood, earthquakes etc. Such losses
are called abnormal losses. These may be divided into two categories:
(i) Loss of Goods : It will be that on the one hand, the loss of goods will
deducted fromthe purchase onthe Debit side of Trading Account and
it will also be shownonthe debit side of Profit &Loss Account
(ii) Loss of Fixed Assets : If some fixed assets of the firm is destroyed by
Bad Debts :
Treatment inFinal Accounts :
Provisions for Bad and Doubtful Debts :
Provisions for Discount on Debtors :
Treatment in Final Accounts :
Provisions for Discount on Creditors :
Abnormal Loss :
174
footer
some accident, then the loss will be shown on the debit side of P&L
A/c and also deducted from the value of Asset on the assets side of
the Balance Sheet.
(15) Occasionally, certain amount of goods is
given away as charity. On the one hand, the amount will be deducted from
purchase and onthe other hand it will also be shown on the debit side of
P&L A/c.
(16) Sometimes the goods which the
business deals inare distributed as free samples for the purpose of
advertising these goods. On the one hand, the amount will be deducted
from purchase and on the other hand it will also be shown on the debit
side of P&L A/c.
(17) If the proprietor of the business has taken some
goods for his personal use from the business, it is known as Drawings in
Goods. It will be deducted from purchase in the Trading Account and will
also be deducted from the Capital on the liabilities side of the Balance
Sheet as Drawings.
(18) There are certain expenditures which
are revenue in nature but the benefit of which is likely to be derived over a
number of years. Such Expenditures are termed as Deferred Revenue
Expenditure. As such, the whole of such expenditure is not debited to the
Profit and Loss Account of the current year but spread over the years for
which the benefit is likely to last. Thus, only a part of such expenditure is
taken to Profit & Loss Account every year and the unwritten off portion is
allowed to stand onthe assets side of the Balance Sheet.
(19) Sometimes, in addition to his
regular salary, the manager is entitled to a commissiononnet profit.
On the one hand, it will be recorded on
the debit side of P& L A/c and on the other hand, shown on the liabilities
side as anoutstanding expense.
(i) OnProfits before charging suchcommission: The formula is:
Rate
Managers Commission= Net Profit x
100
(ii) OnProfits after charging suchcommission: The formula is:
Rate
Managers Commission= Net Profit x
100 + Rate
Charity in the Formof Goods :
Goods Distributed as Free Samples :
Drawings in Goods :
Deferred Revenue Expenditure :
Managers Commission on Net Profit :
Treatment in Final Accounts :
Methods of Calculating the Commission:
175
ACCOUNTING FOR MANAGERS
footer
Q. What is Ratio Analysis? Explain its Objectives and Limitations. Also
give its classification.
:
:
Helpful in Analysis of Financial Statements :
Simplification of Accounting Data :
Helpful in Comparative Study :
Ans. Absolute figures expressed in monetary terms in financial
statements by themselves are meaningless. These figures often do not convey
much meaning unless expressed in relation to other figures. Thus, we can say
that the relationship between two figures, expressed in arithmetical terms is
called a ratio.
Aratio is simply one number expressed in terms of another. It found by dividing
one number into the other.
Ratio Analysis discloses the position of business, so it is a very important tool of
financial analysis. But ratio analysis suffers from a no. of limitations. These
limitations should be kept inmind while making use of the Ratio Analysis.
(1) Ratio analysis is an
extremely useful device for analyzing the financial statement. It helps the
bankers, creditors, investors, shareholder etc. in acquiring enough
knowledge about the profitability and financial healthof the business.
(2) Accounting ratio simplifies and
summarizes a long array of accounting data and makes them
understandable. It discloses the relationship between two such figures
whichhave a cause and effect relationship witheachother.
(3) With the help of ratio analysis
comparison of profitability and financial soundness can be made between
one firm and another in the same industry. Similarly, comparison of
current year figures can also be made withthose of previous years withthe
help of ratio analysis.
Ratio
According to R.N. Anthony
Objectives of Ratio Analysis
ACCOUNTING FOR MANAGERS
MBA 1st Semester (DDE)
UNIT II
176
footer
(4) Current years
ratios are compared with those of the previous years and if some weak
spots are thus located, remedial measures are takento correct them.
(5) Accounting ratios are very helpful in
forecasting and preparing the plans for the future.
(6) If accounting ratios are
prepared for a number of years, they will reveal the trend of costs, sales,
profits and other important facts.
(7) Ratio helps us in establishing ideal
standards of the different items of the business. By comparing the actual
ratios calculated at the end of the year with the ideal ratios, the efficiency
of the business canbe easily measured.
(8) Ratio Analysis discloses the liquidity, solvency and
profitability of the business enterprise. Such information enables
management to assess the changes that have taken place over a period of
time in the financial activities of the business. It helps themin discharging
their managerial functions, e.g. planning, organizing, directing,
communicating and the controlling more effectively.
(9) Ratio analysis discloses the position of
business with different view-points. It discloses the position of business
with the liquidity point of view, solvency point of view, profitability point of
viewetc. With the help of such a study we can drawconclusions regarding
the financial healthof the business enterprise.
1. Accounting ratios are
calculated on the basis of data given in profit &Loss account and balance-
sheet. There are certain limitations of financial statements, and hence the
ratios calculated on the basis of such, financial statements will also have
the same limitation.
2.
There may be different accounting policies adopted by different
firms with regard to providing depreciation etc. For example, one firmmay
Helpful in Locating the Weak Spots of the Business :
Helpful in Forecasting :
Estimate about the Trend of the Business :
Fixation of Ideal Standards :
Effective Control :
Study of Financial Soundness :
:
False accounting date gives false ratios :
Comparison not possible if different firms adopt different accounting
policies :
Limitations of Ratio Analysis
177
ACCOUNTING FOR MANAGERS
footer
adopt the policy of charging dep. On Straight Line Method, while other
may charge on written-down-value method. Such difference makes the
accounting ratios incomparable.
3.
Price level over the years goes on changing, therefore, the ratios of various
years cannot be compared.
4. For e.g. X
Co. produces 10 Lakhmeters of clothin1992 and 15 Lakhmeters in1993,
the progress is 50%. Y Co. raises production from 10 thousand meters in
1992 to 20 thousand meters in 1993, the progress is 100%. Comparison of
these two firms made onthe basis of ratio will disclose that the second firm
is more active that the first firm. Such conclusion is quite misleading
because of the difference in size of the two firms, it is therefore essential to
study the ratios along-withthe absolute data onwhichthey are base.
5. The analyst should not merely rely on a
single ratio. He should study several connected ratios before reaching a
conclusion.
6. Circumstances differ from firm to firm hence
no single standard ratio can be fixed for all the firms against which the
actual ratio may be compared.
7. Ratios derived
from analysis of statements are not sure indicators of good or bad
financial position and profitability of a firm. They merely indicate the
probability of favorable or unfavorable position. The analyst has to carry
out further investigations and exercise his judgment in arriving at a
correct diagnosis.
8. Another important
point to keep in mind is that different persons draw different meaning of
different terms. One analyst persons draw different meaning of different
terms. One analyst may calculate ratios on the basis of profit after interest
and tax, while other may consider profit after interest but before tax
Ratios may be classified into the four categories.
Classificationof ratios canbe explained withthe help of following diagram:
Ratio Analysis becomes Less Effective Due to Price Level Changes :
Ratios may be misleading in the absence of absolute data :
Limited Use of a Single Ratio :
Lack of Proper Standard :
Ratios alone are not adequate for Proper Conclusions :
Effect of Personal ability and bias of the Analyst :
: Classification of Ratios
178
footer
Q. Explain the Important Ratios calculated for Evaluating the Short-
TermSolvency Position of a Company.
OR
Q. Explain the Liquidity Ratios indetail.
: Ans. Liquidity refers to the ability of the firm to meet its
current liabilities. The liquidity ratios, therefore, are also called Short-term
Solvency Ratios. These ratios are used to assess the short-term financial
positionof the concern.
Liquidity Ratios
Classification of Ratios
Liquidity Ratios Leverage Or Activity Or Profitability
Or Short-term Capital Structure Turnover Ratios
Solvency Ratios Ratios Ratios
Current Ratio Stock Turnover Ratio
Liquid Ratio Debtors Turnover Ratio
Average CollectionPeriod
Debt Equity Ratio Creditors Turnover Ratio
Debt to Total Funds Average Payment Period
Ratio Fixed Assets Turnover
Proprietary Ratio Ratio
Fixed Assets to Working Capital
Proprietors fund Ratio Turnover Ratio
Capital Gearing Ratio
Interest Coverage Ratio
Profitability Ratios Profitability Ratios based
based onSales onInvestment
Gross Profit Ratio ReturnonCapital Employed
Net Profit Ratio ReturnonShareholders Fund
Operating Ratio (i) ReturnonTotal
Expenses Ratios Shareholders Funds
(ii) ReturnonEquitys
Shareholders funds
(iii) Earning Per Share
(iv) Dividend per Share
(v) Price Earning Ratio
179
ACCOUNTING FOR MANAGERS
footer
Liquidity ratios include two ratios :
Current Ratio:
Current Assets
Current Ratio =
Current Liabilities
Current Assets :
CONSTITUENTS OF CURRENT ASSETS
CONSTITUENTS OF CURRENT LIABILITIES
1. This ratio explains the relationship between current assets
and current liabilities of a business. The formula for calculating the ratio is:
Current assets include those assets which can be converted
into cashwithina years time.
1. Cash-in-hand and Bank balances
2. Bills Receivables
3. Sundry Debtors (less provision for bad debts)
4. Short-term Loans and Advances
5. Inventories of Stock, as :
(a) Raw materials,
(b) Work-in process
(c) Stores and spares
(d) Finished goods
7. Prepaid Expenses
8. Accrued Incomes
Current Liabilities : All liabilities which are payable within one year are
known as current liabilities.
1. Bills Payables
2. Sundry Creditors or Accounts Payable
3. Accrued or Outstanding Expenses
4. Short-term Loans, Advances and Deposits.
5. Dividends Payables.
6. Bank Overdraft
7. Provision for Taxation, if it does not amount to
appropriation of profits
180
footer
Ideal Ratio :
Significance :
Liquid Ratio :
Liquid Assets
Liquid Ratio = -
Current Liabilities
Liquid Assets :
CONSTITUENTS OF LIQUID ASSETS
OR Liquid Assets= Current Assets- Stock Prepaid Expenses
Ideal Ratio :
According to accounting principle, a current ratio of 2:1 is
supposed to be an ideal ratio. It means that current assets of a business
should, atleast, be twice of its current liabilities. The reason of assuming 2: 1 as
the ideal ratio is that the current assets include such assets as stock, debtors
etc., from which full amount cannot be realized in case of need. Hence, even
half the amount is realized from the current assets on time, the firm can still
meet its current liabilities infull.
This ratio is used to assess the firms ability to meet its short-
termliabilities on time. According to accounting principle, a current ratio of 2:1
is assumed to be an ideal ratio. If the current ratio is less than 2:1, it indicates
lack of liquidity and shortage of working capital. But a much higher ratio, even
though it is beneficial to the short-termcreditors, is not necessarily good for the
company. A much higher ratio than 2:1 may indicate the poor investment
policies of the management. A much higher ratio may be considered to be
adverse from the view point of management on account of the following
reasons:
2. Liquid ratio explains the relationship between liquid
assets and current liabilities of a business. The formula for calculating the
ratio is :
Liquid assets include those assets which will yield cash very
shortly. All current assets except stock and prepaid expenses are included in
liquid assets.
1. Cash-in-hand and Bank balances
2. Bills Receivables
3. Sundry Debtors (less provision for bad debts)
4. Short-term Loans and Advances
5. Temporary Investment of Surplus Funds
6. Accrued Incomes
According to accounting principle, a liquid ratio of 1:1 is
supposed to be an ideal ratio. It means that liquid assets of a business should,
atleast, be equal to its current liabilities. The higher the ratio, the better it is,
because the firmwill able to pay its current liabilities more easily.
181
ACCOUNTING FOR MANAGERS
footer
Significance :
Q. Explain the Important Ratios Calculated for Evaluating the Long -
TermSolvency Position of a Company.
OR
Q. Explain the Capital Structure Ratios indetail
:
Debt Equity Ratio :
Debt Long termLoans
Debt Equity Ratio= OR
Equity Shareholders funds
Debt :
Shareholders Funds :
Significance :
An ideal Liquid ratio is said to be 1:1. If it is more, it is
considered to be better. The idea is that for every rupee of current liabilities,
there should atleast be one rupee of liquid assets. This ratio is a better test of
short-term financial position of the business other than the current ratio, as it
considers only those assets which can be easily and readily converted into
cash. Liquid ratio thus is a more rigorous test of liquidity than the current ratio
and, whenused together withcurrent ratio, it gives a better picture of the short-
termfinancial positionof the business.
Ans. These ratios are calculated to assess the
ability of the firm to meet its long term liabilities when they become due. Long
term creditors including debenture holder and primarily interested to know
whether the co. has ability to pay regular interest due to themand to repay the
principal amount when it become due. These ratios includes the following
ratios:-
These ratios include the following:
1.
These refer to long-term liabilities which mature after one year. These
include Mortgage Loan, Debenture, Bank Loan, Loan from financial
institutions, Public Deposits etc.
Equity Share Capital, Preference Share capital,
Securities premium, General Reserve, Capital Reserve, other reserves and
credit balance of profit &loss a/c.
However, accumulated losses and fictitious assets remaining to the written off
like preliminary expenses, underwriting commission, share issue expense etc,
should be deducted.
This ratio is calculated to assess the liability of the firmto meet
its long-term liabilities. Generally, debt equity ratio of 2:1 is considered safe. If
the debt equity ratio is more that that, it shows a rather risky financial position
from the long term point of view, as it indicates that more and more funds
invested business are provided by long-termlenders. A high debt equity ratio is
a danger-signal for long-termlenders.
Capital Structure Ratios
182
footer
2.
Generally, debt to total fund ratio is (.67:1) is considered
satisfactory. In other words, the proportion of long termloans should not more
than 67%of total funds. A high ratio than this is generally treated an indicator
of risky financial position from the long-term point of view, because it means
that the firmdepends too muchuponoutside loans for its existence.
3.
Equity
Proprietary Ratio =
Equity + Debt
This ratio should be 33%or more than that. In other words, the
proportion of shareholders funds to total funds be 33% or more. A higher
proprietary ratio is generally treated an indicator of sound financial position
fromlong-termpoint of view.
4.
Fixed Assets
Fixed Assets to Proprietors Ratio=
Proprietors funds (net worth)
The ratio indicates the extent to which proprietors funds are
sunk into fixed assets. Normally, the purchase of fixed assets should be
financed by proprietors funds. If this ratio is less than 100%, it would mean
that proprietors funds are more than fixed assets and a part of working capital
is provided by the proprietors.
5.
Significance : A high gearing will be beneficial to equity shareholders when the
rate of interest/dividend payable on fixed cost bearing capital is lower than the
rate of returnoninvestment inbusiness.
Debt to Total Funds Ratio :
Significance :
Proprietary Ratios :
Significance :
Fixed Assets to Proprietors Ratio :
Significance :
Capital Gearing Ratio :
Fixed Cost bearing capital = Preference share capital+ Debenture+ Long
termloans
Debt Long termloans
Debt to total funds ratio= OR
Debt+ Equity long termloans+ shareholders Funds
Equity Share Capital+ Reserves + P&L (Cr.) Balance
Capital Gearing Ratio=
Fixed Cost bearing capital
183
ACCOUNTING FOR MANAGERS
footer
6.
Net Profit before interest &tax
Interest Coverage Ratio =
Fixed Interest Charges
This ratio indicates how many times the interest charges are
covered by the profits available to pay interest charges. A long term lenders in
finding out whether the business will earn sufficient profits to pay the interest
charges regularly. The higher ratio more secure the lender is in respect of
payment of interest regularly. An interest coverage ratio of 6 to 7 times is
considered appropriate.
Ans. These ratios are calculated on the basis of cost of sales
or sales, therefore, these ratios are also called as Turnover Ratios. Turnover
indicates the speed or number of items the capital employed has been rotated
in the process of doing business. In other words, these ratios indicated how
efficiently the capital is being used to obtain sales; how efficiently the fixed
assets are being used to obtain sales; and how efficiently the working capital
and stock is being used to obtain sales. Higher turnover ratios indicate the
better use of capital or resources and in turn lead to higher profitability.
Turnover ratios include the following:
1) This ratio indicates whether inventory has
been efficiently used or not. This ratio indicates the relationship between
the cost of goods sold during the year and average stock kept during that
year. The formula for calculating the ratio is :
Cost of goods sold canbe calculated by two ways :
Cost of Goods Sold = Sales Gross Profit
OR
Cost of Goods Sold = Opening Stock + Purchases + Carriage + Wages
+ Other Direct Expenses Closing Stock
This ratio shows the speed with which the stock is rotated into
sales or the number of times the stock is turned into sales during the year. The
higher the ratio, the better it is, since it indicates that stock is selling quickly. In
Interest Coverage Ratio :
Significance :
Q. Explain the Activity Ratios Or Turnover Ratios indetail.
:
Inventory Turnover Ratio :
Cost of Goods Sold
Inventory Turnover Ratio =
Average Stock
Opening Stock + Closing Stock
Average Stock =
2
Significance :
Activity Ratios
184
footer
a business where stock turnover ratio is high, goods can be sold at a lowmargin
of profit and eventhenthe profitability may be quite high.
(2) This ratio indicates the time within which the
stock is converted into sales. This ratio is computed by the following
formula:
Inventory holding period canbe calculated indays or months or weeks.
(3) This ratio indicates the relationship between
credit sales and average debtors during the year. The formula for
calculating the ratio is:
Net Credit Sales = Total Sales CashSales
Bills receivable are added in debtors for the purpose of calculation of this ratio.
While calculating this ratio, provision for bad and doubtful debts is not
deducted from total debtors, so that it may not give a false impression that
debtors are collected quickly. Debtors turnover ratio can be calculated on the
basis of total sales instead of credit sales.
This ratio indicates the speed with which the amount is
collected from debtors. The higher the ratio, the better it is, since it indicates
that amount from debtor is being collected more quickly. The more quickly the
debtors pay, the less the risk from bad debts, and so the lower the expenses of
collectionand increase inthe liquidity of the firm.
(4) This ratio indicates the time within which
the amount is collected from debtors and bills receivable. This ratio can be
computed by the following three formulas:
Inventory Holding Period :
12months/ 52 weeks/ 365 days
Inventory Holding Period =
Stock Turnover Ratio
Net Credit Sales
Debtors Turnover Ratio =
Average Debtors + Average B/R
Opening Debtors + Closing Debtors
Average Debtors =
2
Opening B/R+ Closing B/R
Average B/R=
2
Significance :
Average Collection Period :
Debtors Turnover Ratio :
185
ACCOUNTING FOR MANAGERS
footer
First Formula :
Average Debtors + Average B/R
Average CollectionPeriod =
Credit Sales per day
Net Credit Sales of the Year
Credit Sales per Day =
365
:
Average Debtors x 365
Average CollectionPeriod =
Net Credit Sales
Third Formula :
12 months/ 365 days/ 52 weeks
Average CollectionPeriod =
Debtor Turnover Ratio
Significance :
Creditors Turnover Ratio :
Net Credit Purchases
Creditors Turnover Ratio =
Average Creditors + Average B/P
Net Credit Purchase = Total Purchases CashPurchase
Opening Creditors + Closing Creditors
Average Creditors =
2
Opening B/P + Closing B/P
Average B/P =
2
This ratio shows the time in which the customers are paying for
credit sales. For example, in a business average collection period is 30 days. It
means that, on an average, if sale is made today, the cash will be collected
actually after 30 days, i.e., 30 days credit sales are locked up indebtors.
(5) This ratio indicates the relationship between
credit purchases and average creditors during the year. The formula for
calculating the ratio:
Second Formula
186
footer
This ratio can be calculated on the basis of total purchases instead of credit
purchases.
This ratio indicates the speed with which the amount is being
paid to creditors. The higher the ratio, the better it is, since it will indicate that
the creditors are being paid more quickly which increases the credit worthiness
of the firm.
(6) This ratio indicates the time which is
normally taken by the firmto make payment to its creditors. This ratio can
be calculated by the following three formulas:
This ratio shows the time in which the creditors are paid for
credit purchases. The lower the ratio, the better it is, because a shorter
payment period implies that the creditors are being paid rapidly.
(7) This ratio indicates the relationship
between cost of goods sold and working capital. The formula for
calculating the ratio is:
This ratio indicates how efficiently working capital has been
utilised in making sales. This ratio is of particular importance in non-
manufacturing concerns where current assets play a major role in generating
sales. This ratio shows the number of times on which working capital has been
Significance :
Average Payment Period :
First Formula :
Average Creditors + Average B/P
Average Payment Period =
Credit Purchase per day
Second Formula :
Average Creditors x 365
Average Payment Period =
Net Credit Purchases
Third Formula :
12months/ 52 weeks/ 365 days
Average Payment Period =
Creditors Turnover Ratio
Significance :
Working Capital Turnover Ratio :
Cost of Goods Sold
Working Capital Turnover Ratio =
Working Capital
Working Capital = Current Assets Current Liabilities
Significance :
187
ACCOUNTING FOR MANAGERS
footer
rotated in producing sales. A high working capital turnover ratio shows
efficient use of working capital and quick turnover of current assets like stock
and debtors.
Ans. The main object of every business is to earn profits.
A business must be able to earn adequate profits in relation to the risk and
capital invested in it. The efficiency and the success of a business can be
measured with the help of profitability ratios. Profitability Ratios can be
determined onthe basis of either sales or investment into business.
(A) These ratios include the following
(1) This ratio shows the relationship between gross
profit and sales. The formula for computing this ratio is:
Gross Profit = Sales Cost of Goods Sold
Net Sales = Sales Sales Return.
Significance : This ratio measures the margin of profit available on sales. The
higher the gross profit ratio, the better it is. No ideal standard is fixed for this
ratio, but the gross profit ratio should be adequate enough not only to cover the
operating expenses but also to provide for depreciation, interest on loans,
dividends and creationof reserves.
(2) This ratio shows the relationship between net profit
and sales. It may be calculated by two methods:
(i)
Net Profit= Gross Profit- All Indirect Expenses + All indirect Incomes
(ii)
Q. Explain the Important Ratios Calculated for evaluating the
Profitability of a Company.
OR
Q. Explain the Profitability Ratios indetail
:
:
Gross Profit Ratio :
Gross Profit
Gross profit Ratio= x 100
Net Sales
Net Profit Ratio :
Net Profit Ratio :
Net Profit
Net Profit Ratio= x100
Net Sales
Operating Net Profit Ratio :
Operating Net Profit
Operating Net Profit Ratio = x100
Net Sales
Profitability Ratios
Profitability Ratios Based on Sales
188
footer
Operating Net Profit= Gross Profit- Operating Expenses
Operating Expenses= Office and Administration Expenses, Selling and
distributionexpenses, Bad debts, Discount, Interest onshort-termdebts.
This ratio measures the rate of net profit earned on sales. It
helps in determining the overall efficiency of the business operations. An
increase in the ratio over the previous year shows improvement in the overall
efficiency and profitability of the business.
(3) This ratio measures the proportion of an enterprises
cost of sales and operating expenses incomparisonto its sales:
Cost of Goods Sold + Operating Expenses
Operating Ratio : X100
Net Sales
Cost of Goods Sold = Sales Gross Profit
OR
Cost of Goods Sold = Opening Stock + Purchases + Carriage + Wages +
Other Direct Expenses Closing Stock
= Office and Administration Expenses, Selling and
distributionExpenses, Bad debts, Discount, Interest onshort-termdebts
Operating ratio is a measurement of the efficiency and
profitability of the business enterprise. The ratio indicates the extent of sales
that is absorbed by the cost of goods sold and operating expenses. Lower the
operating ratio, the better it is, because it will leave higher margin of profit on
sales.
(4) These ratios indicate the relationship between
expenses and sales. The ratio may be calculated as:
(i)
(ii)
(iii)
Significance :
Operating Ratio :
Cost of goods sold can be calculated by two ways :
Operating Expenses
Significance :
Expenses Ratios :
Material Consumed
Material Consumed Ratio = X100
Net Sales
Direct Labour Cost
Direct Labour Cost Ratio = X100
Net Sales
Factory Expenses
Factory Expenses Ratio = X100
Net Sales
189
ACCOUNTING FOR MANAGERS
footer
(B) These
ratios reflect the true earning capacity of the resources employed in the
enterprise Sometimes the profitability ratios based on sales are high
whereas profitability ratios based on investment are low. These may be
classified into two categories:
(1) ReturnonCapital Employed
(2) ReturnonShareholders Funds
(1) This ratio reflects the overall profitability
of the business. This ratio is also known as Rate of Return or Yield on
Capital. The ratio is computed as under:
This can be computed by any of the following two
methods:
Capital Employed = Debt + Equity NonOperating Assets
OR
Capital Employed = Fixed Assets + Current Assets Current Liabilities
(2) Return on shareholders funds
measures only the profitability of the funds invested by shareholders.
There are several measures to calculate the return on shareholders
funds:
(i) The ratio is computed as under:
This ratio reveals how profitably the proprietors funds have
been utilized by the firm. A comparison of this ratio with that of similar firms
will throwlight onthe relative profitability and strengthof the firm.
(ii) This ratio is computed as
under:
Profitability Ratios Based on Investment in the Business :
Return on Capital Employed :
Capital Employed :
Return on Shareholders Funds :
Return on Total Shareholders Funds :
Total Shareholders Funds = Equity Share Capital + Preference Share
Capital + All Reserves + P&L A/c Balance Fictitious assets
Significance :
Return on Equity Shareholders Funds :
Profit before Interest, tax and dividends
Return on Capital Employed = X100
Capital Employed
Net profit After Interest and Tax
Return on Total Shareholders Funds = X100
Total Shareholders Funds
Net profit After Interest, Tax and Preference Dividend
Return on Equity Shareholders Funds = X100
Equity Shareholders Funds
190
footer
Equity Shareholders Funds = Equity Share Capital + All Reserves + P&L
A/c Balance Fictitious Assets
This ratio measures how efficiently the equity shareholders
funds are being used inthe business.
(iii) This ratio measures the profit available to
the equity shareholders ona per share basis. This ratio is computed as under:
This ratio is helpful in the determination of the market price of
the equity share of the company.
(iv) Profit remaining after payment of tax and
preference dividend are available to equity shareholders. But all of these
are not distributed among themas dividend. Out of these profits, a portion
is retained in the business and remaining is distributed among equity
shareholders as dividend.
(v) This ratio is computed as under:
(vi)
(vii) Price Earnings(P.E) Ratio :
Significance :
Earning Per Share (E.P.S.) :
Net Profit Dividend on Preference Shares
Earning Per Share =
Number of Equity Shares
Significance :
Dividend Per Share :
Dividend Paid to Equity Shareholders
Dividend Per Share =
Number of Equity Shares
Dividend Payout Ratio Or D.P. :
D.P.S
D. P. = X100
E.P.S
Earning and Dividend Yield :
EPS
Earnings Yield = X100
Market Value Per Share
DPS
Dividend Yield = X100
Market Value Per Share
Market price of the share
P.E. Ratio =
EPS
191
ACCOUNTING FOR MANAGERS
footer
Q. What is Fund FlowStatement? Howis it prepared?
:
Meaning of Funds :
Meaning of Flow :
:
Schedule of Changes in Working Capital :
SCHEDULE OF CHANGES IN WORKING CAPITAL
Ans. The balance sheet of a firmdiscloses
the position of assets, liabilities and capital at the end of a particular year. But
it does not disclose the causes of changes in these items between the end of
previous year and the end of current year. Therefore, an additional statement
called Fund Flow Statement is prepared to show the changes in assets,
liabilities and capital betweenthe dates of two balance sheets.
In a limited sense, the term fund means cash. But this
is not the correct meaning of the term fund because there are many
transactions inthe business whichdo not result ininflowor outflowof cashbut
certainly result inthe inflowor outflowof funds. As such, the termfund stands
for Net Working Capital.
The termflow means change or movement. Therefore, the
term Flow of Funds means increase or decrease in working capital. If a
transaction results in the increase of working capital, it is said to be a source of
funds and if the transaction results in the decrease of working capital, it is said
to be an application of funds. If the transaction does not result in any change in
the working capital, it is said that it does not result inthe flowof fund.
For preparing Fund Flow Statement
we have to prepare the following three statements:
(1) This schedule considers only
current assets and current liabilities, at the beginning and at the end of
the year. This schedule shows either increase or decrease in working
capital.
Meaning of Fund FlowStatement
Preparation of Fund Flow Statement
Particulars Amount Amount Increase in Decrease in
As on As on Working Working
Capital Capital
Cash-in-hand
Cash at Bank
Debtors
Closing Stock
Short Term
investments
Bills Receivables
Prepaid Expenses
Other current assets ________
.. ..
________
Current Assets:

192
footer
Bank Overdraft
Bills Payable
Creditors
Provision for Taxation
Proposed Dividend
O/s Expenses
Unclaimed Dividend
Working Capital (Current
Assets Current Liabilities)
Net Increase or Decrease
in Working Capital
Total Current Assets
Current Liabilities:
Total Current Liabilities

________ ________ ________ ________


________ ________ ________ ________
(2) In order to prepare a funds
flow statement it is necessary to ascertain the sources and application of
funds. Main source of fund in a business is funds from operations
Net Profit As per Profit & Loss A/c
(i) Depreciation.
(ii) Goodwill written off.
(iii) Preliminary Expenses.
(iv) Patent Rights, Trade Marks and Copy Rights.
(v) Discount on issue of Debentures & Shares.
(vi) Deferred Revenue Expenditure such as,
Advertisement Suspense A/c.
(i) Loss on sale of Fixed Assets
(ii) Appropriation of Profit:
Transfer to General Reserve
Transfer to Sinking Fund
Transfer to Dividend
Equalisation Fund etc.
(i) Proposed Dividend
Calculation of Funds from Operations :
STATEMENT SHOWING FUNDS FROM OPERATIONS
Particulars Amount
(A) Items to be Added back to Net Profit:
(a) Non-Fund Items:
(b) Non- Trading Losses:
193
ACCOUNTING FOR MANAGERS
footer
(ii) Provision for Taxation
(i) Profit on sale of Fixed Assets
(ii) Receipt of Dividend
(iii) Re-Transfer of Excess Provisions
Fund Flow Statement is prepared to show the
changes in assets, liabilities and capital between the dates of two balance
sheets. It discloses the causes of changes in the items of balance sheet
between the end of the previous year and the end of current year. Thus, by
preparing this statement, the management can find out the basic reasons
for changes in the assets, liabilities and capital of the firm between two
balance sheets.
1. Funds from Operations 1. Loss from Operations
2. Issue of Shares 2. Buy back of Equity shares and
Redemption of Preference Shares
3. Issue of Debentures 3. Redemption of Debentures
4. Raising Long-Term Loans 4. Repayment of Long-Term Loans
5. Sale of Fixed Assets 5. Purchase of Fixed Assets
6. Non-Trading Receipts 6. Non-Trading Payments
Ans. The balance sheet of a firmdiscloses
the position of assets, liabilities and capital at the end of a particular year. But
it does not disclose the causes of changes in these items between the end of
previous year and the end of current year. Therefore, an additional statement
called Fund Flow Statement is prepared to show the changes in assets,
liabilities and capital betweenthe dates of two balance sheets.
(B) Items to be Deducted from Net Profit
Fund Flow Statement :
FORMATOFFUNDFLOWSTATEMENT:
FUND FLOW STATEMENT
Sources of Funds Applications of Funds
Q. What is Fund Flow Statement? What are the uses and Limitations of
Fund FlowStatement?
:
:

Meaning of Fund FlowStatement


Uses of Fund FlowStatement
194
footer
1.
Afund flowstatement is prepared to give satisfactory answer
to the following questions:
(i) What have been the main sources and applications of funds during
the period?
(ii) Howmuchfunds have beengenerated frombusiness operations?
(iii) Where did the profits go?
(iv) Why are dividends not larger?
(v) Howwas the expansioninplant and Equipment financed?
(vi) Howwas the repayment of long termdebt accomplished?
(vii) Howwas the increase inWorking Capital financed?
2. A fund flow statement provides a
complete analysis of the financial position of a firm. This objective is not
achieved by the balance sheet because it gives a static viewof the financial
position of a business by showing the assets and liabilities at a particular
point of time.
3. A
fund flow statement gives a much more reliable figure of the profits of the
business than the figures shown by profit and loss account is affected by
the personal decisions of management in deciding the amount of
depreciation and other adjustments regarding the writing off of
preliminary expenses etc.
4. The
fund flow statement enables the management to know whether the funds
have been properly used in purchasing various assets or repaying loans
etc.
5. While managing
working capital ina business, it becomes essential to ensure that it should
neither be excessive nor inadequate. A fund flow statement indicates the
excessiveness or inadequacy inworking capital.
6. If a fund flow
statement is prepared for next year, it will enable the management to plan
its financial resources properly. The firm will know how much funds it
requires, how much the firm can manage internally and how much it
should arrange from outside source. This is helpful in preparing the
budgets for the future period.
7. Sometimes, there may be
sufficient profits but the distribution may not be possible due to its
adverse effect onthe liquidity and working capital of the business.
Helpful in Finding the Answers to Some Important Financial
Questions :
Helpful in Financial Analysis :
It provides more reliable figures of profit and loss of the business :
It enables to know whether the funds have been properly used :
Helpful in proper Management of Working Capital :
Help in the presentation of Budget for the next period :
Helpful in Determining Dividend Policy :
Limitations of Fund FlowStatement :
195
ACCOUNTING FOR MANAGERS
footer
1. Fund flowstatement ignores certainnon-fund transactions.
2. It reveals only the changes in working capital and does not show the
changes incashposition.
3. It is historical innature because it reports what has happened inthe past.
4. Since it is based on opening and closing balance sheets and the profit and
loss account, it is not anoriginal statement.
Ans. A cash-flow statement is a statement showing
inflows and outflows of cash during a particular period. In other words, it is a
summary of sources and applications of cash during a particular span of time.
It analyss the reason for changes in balance of cash between the two balance
sheet dates. The termcash here stands for cash and cash equivalents. A cash-
flowstatement canbe for the past or canbe projected for a future period.
A cash flow statement may be
prepared either by direct or indirect method. Format under indirect method is
givenbelow:
Net profit before Tax .............
Add : Non-Cash Expenses
: Non-operating Expenses
Operating Profit before Working Capital Changes ________
Add : Decrease in Current Assets
: Increase in Current Liabilities
________
Less : Increase in Current Assets (
: Decrease in Current Liabilities (
________
Cash generated from operating activities
Income Tax paid (
Q. What is Cash Flow Statement? Give the Format of Cash Flow
Statement Or Howis it prepared?
:
:
XYZ LTD.
CASH FLOW STATEMENT for the year ending
(Indirect Method)
Particulars Amount Amount
(A) CashFlows fromOperating Activities :
Cash Flow Statement
FORMAT OF CASH FLOW STATEMENT
.............
.............
.............
.............
.............)
.............)
.............
.............)
196
footer
________
Cash flows before extraordinary items
(+) or (-) Extraordinary items
Purchase of fixed assets (
Sale of fixed assets
Purchase of investments (long-term) (
Sale of Investment (long-term)
Interest received
Dividend received
Proceeds from issue of share capital
Proceeds from long-term borrowings
Repayments of long-term borrowings (
Interest paid (
Dividend paid (
Net Increase or decrease in cash and cash
equivalents ( A+B+C)
Cash and cash equivalent at the beginning
of the period ________
Ans. A cash-flow statement is a statement showing
inflows and outflows of cash during a particular period. In other words, it is a
summary of sources and applications of cash during a particular span of time.
It analysis the reason for changes in balance of cash between the two balance
sheet dates. The termcash here stands for cash and cash equivalents. A cash-
flowstatement canbe for the past or canbe projected for a future period.
.............
.............
.............
.............)
.............
.............)
.............
.............
.............
.............
.............
.............
.............)
.............)
.............)
.............
________
.............
.............
.............
Net Cash from Operating Activities ________
(B) Cash Flows from Investing Activities :
Net Cash from Investing Activities ________
(C) Cash Flows from Financing Activities:
Net Cash from Financing Activities _________
Cash and Cash equivalent at the end
of the period ________
Q. What is Cash Flow Statement. Discuss Its main Uses and
Limitations?
:
:

Cash Flow Statement


Uses of CashFlowStatement
197
ACCOUNTING FOR MANAGERS
footer
1. A cash flow statement
provides information for planning the short-term financial needs of the
firm. Since it provides information regarding the sources and utilization of
cash during a period, it becomes easier for the management to assess
whether it will have adequate cash to meet day-to-day expenses and pay
the creditors intime.
2. A cash flow statement prepared
for the future period in helpful in preparing a cash budget. It informs the
management about the surplus or deficit periods of cash. It helps in
planning the investment of surplus cash in short-terminvestments and to
planshort-termcredit inadvance for deficit period.
3. A cash flow
statement reveals the speed at which the cash is being generated from
debtors, stock and other current assets and the speed at which the
current liabilities are being paid. It enables the management to assess the
true positionof the cashinnature.
4. A firmmay earn huge
profits yet it may have paucity of cash or when it suffered a loss it may still
have plenty of cash. Acashflowstatement explains the reasons for it.
5. Dividend must be paid within
42 days of its declaration. Hence the management takes the help of cash
flow statement to ascertain the position of cash generated from operating
activities whichcanbe used for payment of dividend.
6. A cash-flow
statement reveals the speed at which the cash is being generated from
debtors, stock and other current assets and the speed at which the
current liabilities are being paid. It enables the management to assess the
true positionof the cashinfuture.
It does not present true picture of the liquidity of a firm because the
liquidity does not depend uponcashalone.
The possibility of window-dressing is higher in case of cash position in
comparisonto the working capital positionof a firm.
Cashflowstatement ignores non-cashcharges.
It is prepared on cash basis and hence ignores one of the basis concepts of
accounting, namely accrual concept.
Ans.
Useful for short-term financial planning :
Useful in preparing the Cash Budget :
Study of the trend of cash receipts and payments :
It explains the deviation of cash fromearnings :
Helpful in making Dividend Decisions :
Study of the Trend of Cash Receipts and Payments :
Limitations of CashFlowStatement :
1.
2.
3.
4.
Q. Distinctionbetween Fund FlowStatement and CashFlowStatement.
: Distinctionbetween Fund FlowStatement and CashFlowStatement
198
footer
1. Basis of It discloses the causes of It discloses the
changes in working capital. cause of changes
in cash position.
2. Interpretation Sound funds position doesnt Sound cash position
mean sound cash position means sound funds
because inflow of funds position as inflow of
doesnt necessarily involve cash necessarily
inflow of cash. involves inflow
of funds.
3. Difference in In case of fund flow analysis, In case of cash flow
the method of an increase in a current analysis an
preparation liability or decrease in a increase or
current assets results in decrease in a
decrease in working capital current assets
& vice-versa. results in increase
in cash &
vice-versa.
4. Usefulness It is more useful for long-term It is more useful for
financial planning. short-term financial
planning.
5. Schedule of Schedule of changes in No separate
Changes in working capital is also schedule is
Working Capital preparing to study the prepared with the
changes in current assets cash flow statement
and current liabilities. as the changes in
current assets and
current liabilities
are shown by way of
adjustment in profit
to arrive at cash
flow from operating
activities.
6. Opening & Opening & closing balance Opening & closing
closing Balance of cash are not shown in fund balance of cash are
of Cash flow statement, as they are shown in this
shown in schedule of changes statement
in working capital.
BASIS FUND FLOW CASHFLOW
STATEMENT STATEMENT
Analysis
199
ACCOUNTING FOR MANAGERS
footer
7. Principles of This is prepared on accrual This is prepared on
Accounting basis of accounting cash basis of
accounting.
Ans.
1. Nature It discloses the sources and It discloses net
uses of funds. profit or net loss
made during
the year.
2. Items All items whether capital or Only revenue items
revenues are considered in are considered in
preparing this statement. this statement.
For Example:- Cash received For Example:- Cash
from shares issue is received from share
considered a source of funds issue is not
considered an
income.
3. Objects It helps in the financial It is prepared to
management of the business ascertain the net
results of business
operations.
4. Optional or Preparation of fund flow Preparation of
Compulsory statement is optional and if it income statement is
is prepared, it can be one so compulsory and the
in any manner. contents of income
statement must be
in accordance with
requirements of law.
5. Help Income statements help the Funds flow
preparation of funds flow statement doesnt
statement because funds from help the
operations are found out from preparation of
income statement. income statement.
Q. Distinction Between Fund FlowStatement And Income Statement Or
Profit And Loss Account.
BASIS FUND FLOW STATEMENT INCOME
STATEMENT
Distinction Between Fund FlowStatement And Income Statement Or
Profit And Loss Account
200
footer
6. Scope Its scope is limited because it It is of utmost
only shows changes in important because
working capital, i.e. sources it depicts the result
and uses of funds. of all financial
transactions of a
particular period.
Ans.
BASIS FUNDFLOWSTATEMENT BALANCESHEET
1. Nature It discloses the sources and It is a statement of
uses of funds assets and liabilities
at appoint of time.
2. Items All items whether capital or Only the items of
revenues are considered in assets, liabilities
preparing this statement. and capital are
considered in
preparing this
statement.
3. Use It is a tool of management Not only the
techniques and is used by the management but
management for future some external
financial planning and budget parties are also
formation. interested in it
because it is
prepared to reveal
the financial
position of the firm.
4.Optional or Preparation and publication of Preparation and
Compulsory fund flow statement is optional publication of
and if it is prepared, it can be Balance Sheet is
done in so in any form. compulsory and it
is prepared in its
prescribed form.
Q. DistinctionBetween Fund FlowStatement And Balance Sheet.
: DistinctionBetween Fund FlowStatement And Balance sheet
201
ACCOUNTING FOR MANAGERS
footer
5. Basis of It is prepared with the help of It is prepared with
preparation two balance sheet and profit & the help of trial
loss A/c. balance.
6. Scope Its scope is limited because it It is so important
only shows changes in because it shows
working capital, i.e. sources the financial
and uses of funds. position of the
business.
7. Term It is prepared to know the It is an yearly affair
temporary changes throughout the
whole life of the
business.
202
footer
Q. What is Management Accounting? Explain its Nature, Importance
and Roles.
:
:
:
According to Robert. N. Anthony
:
It lays more Emphasis on Future :
Ans. The main objective of financial accounting is to provide
information about the profitability and financial position of an enterprise by
preparing trading and profit &loss account and a balance sheet. But it does not
present the accounting information in such a way to assist the management in
planning day-to-day operations of a business and to make various types of
decisions. There are various limitations of financial accounting and
management accounting removes these limitations.
Management Accounting is
comprises of two words Management and Accounting. It is the study of
managerial aspect of accounting. The emphasis of management accounting is
to redesign accounting in such a way that it is helpful to the management in
formation of policy, control of execution and appreciation of effectiveness. It is
that system of accounting which helps management in carrying out its
functions more efficiently. Management Accounting presents the financial data
in such a way as to assist the management in planning and controlling the
activities of the firm. Management Accounting is also known as accounting for
management.
Management Accounting is concerned with accounting information that
is useful to management.
According to Institute of Chartered Accountants of England and Wales
Any form of accounting which enables a business to be conducted more
efficiently canbe regarded as management accounting.
1. Management accounting is
concerned with the future. It helps the management in forecasting and
planning the future course of action.
Introduction
Meaning of Management Accounting
Definitionof Accounting
Nature of Management Accounting
ACCOUNTING FOR MANAGERS
MBA 1st Semester (DDE)
UNIT III
203
footer
2. It is a technique of selective Nature. It
takes into consideration only that data from the profit and loss account
and balance sheet which is relevant and useful to the management. Only
that information is communicated to the management which is helpful to
decisionmaking.
3. The cause and effect
relationship is studied in management accounting. For Example, if the
profits are lower than expectations, the reason for the same are
investigated. On the other hand, if the profits are more than expectations,
reasons for the higher profitability are analysed. The effect of various
decisions such as pricing, promoting a new product, sales mix, cost
control etc. is studied onthe profitability of the business.
4. The management
accountant never takes any decisions but only provides data on the basis
of whichthe management takes decisions.
5. The management
accountant uses various techniques and concepts to make the accounting
data more useful for managerial decisionmaking.
6. No specific rules are followed in
management accounting. It provides information in the form which may
be more suitable to the management intaking various decisions.
Management accounting is very
important because it enables management to maximize profits or minimize
losses. Its importance is as follows:-
1. Management accounting supply
information to the management for formulating plans. Planning is
essentially related to taking decisions for future. It also includes
forecasting setting goals and deciding alternative courses of action. So
management Accounting is helpful inplanning and policy formation.
2. Management accounting devices
like standard costing & budgeting controls are helpful in controlling
performance. The work is divided in to different units &separate goals are
set up for each unit. The management accounting act as a co. ordinating
link between different departments and he also monitors the performance
of top management.
3. Organisation is related to the establishment of
relationship among different individuals in the concern. It also includes
delegating of authority and fixing of responsibility. All these aspects are
set up by management accounting.
Techniques of selective Nature :
It establishes cause and effect Relationships :
It Provides Information and not the Decisions :
Use of Special Techniques and Concepts :
No Set Rules and Formats :
:
Planning and Policy Formation :
Helpful in controlling performance :
Helpful in Organizing :
Importance of Management Accounting
204
footer
4. the main object of
management accounting is present financial information to them in such
away that it is easily understood. Management accounting explains all the
statement to the management in simple language. If necessary he uses
statistical devices like charts, diagrams, index number etc. so that the
informationeasily followed.
5. Management accounting helps in
coordinating the activities of different persons and different departments
by establishing a management informationsystem
6. Management accounting helps the
management in selecting best alternatives of doing the things. Targets are
laid down for the employees. They feel motivated in achieving their targets
and further incentives may be givenfor improving their performance.
7. The management has to take certain
important decisions. A decision may have to be taken about the expansion
or diversification of production. The information provided by the
accounting helps the management in selecting a suitable alternative and
taking correct decision.
8. One of the primary importances of
management accounting is to keep the management fully inform about
the latest information of the concern. This helps the management taking
proper & timely decisions. Management accounting is a new approach to
accounting. It provides techniques for the interpretation of accounting
data. The following facts of management accounting are of a great
significance and formthe scope of this project
9. Management accounting helps to increase the
efficiency of management. The targets of various departments are fixed
and communicated to them well in advance. This helps in increasing the
efficiency of the management.
10. Management accounting measures the
profitability of each activity and each department. Only those activities
and departments are continued which promise better results.
Unprofitable activities and departments are discontinued. It maximizes
the profitability of the organization.
The functions of management
accountant depend upon his status in the organisation, needs of the enterprise
and personal capabilities of the persons. So role of management is very
necessary for every organisation. Its role is as follows:-
1. Management Accountant establishes, co-
Helpful in interpreting financial information :
Helpful in Coordination :
Motivating employees :
Helpful in making decisions :
Reporting to management :
Increases Efficiency :
Maximizing Profitability :
Role of Management Accounting :
Planning for control :
205
ACCOUNTING FOR MANAGERS
footer
ordinates and maintains an integrated plan for the control of operations.
Such a plan would provide cost standards, expenses budgets, sales
forecasts etc.
2. Management accounting measures performance against
given plans and standards. The result of operations is interpreted to all
levels of management. This functionwill include installationof accounting
& costing systems and recording of actual performance so as to find out
deviation(if any).
3. Management accountant should evaluate various policies
and programmes. The effectiveness of planning and procedures to attain
the objectives of the organisation will depend upon the caliber of the
management accountant.
4. Management accountant is expected to report
to govt. agencies as required under different laws and to supervise all
matters relating to taxes.
5. He is to assess the effective various
economic and fiscal policies of the govt. and also evaluate the impact of
other external factors onthe attainment of organisational objects.
6. The protection of business assets another
function assigned to the management accountant. This function is
performed through the maintenance of internal control, auditing and
assuring proper insurance courage of assess.
Ans. Management Accounting is
comprises of two words Management and Accounting. It is the study of
managerial aspect of accounting. The emphasis of management accounting is
to redesign accounting in such a way that it is helpful to the management in
formation of policy, control of execution and appreciation of effectiveness. It is
that system of accounting which helps management in carrying out its
functions more efficiently. Management Accounting presents the financial data
in such a way as to assist the management in planning and controlling the
activities of the firm. Management Accounting is also known as accounting for
management.
Management Accounting is concerned with accounting information that
is useful to management.
Scope of management accounting
includes all those activities which are helpful in the collection and analysis of
information.
Reporting :
Evaluating :
Administration of Tax :
Appraisal of external effects :
Protection of Assets :
Q. Explain Management Accounting. Describe the Scope Of
Management Accounting.
:
Definitionof Accounting :
Robert. N. Anthony
:
Meaning of Management Accounting
Scope of Management Accounting
206
footer
Scope of Management Accounting
Internal
Audit
Financial
Accounting
Cost
Accounting
Financial
Management
Budgeting &
Forecasting
Tax
Accounting
Inventory
Control
Interpretation
of data
1. Financial accounting deals with the historical
data the recorded fact about an organisation are use for planning the
future course of action. Though planning is always for future but still it
has to be based on past and present data. The control aspect too is based
on financial data. The performance appraisal is based on recorded facts
and figures. So management accounting is closely related to financial
accounting.
2. cost accounting provides various techniques for
determining cost of manufacturing products or cost of providing service.
It uses financial data for finding out cost of various jobs, products or
process. The system of standard costing, marginal costing etc. are all
helpful to management for planning various business activities.
3. It is concerned with planning and controlling
of the financial resources of a firm. It deals with raising of funds and their
effective utilization. Its main aim is to use business funds in such a way
that earnings are maximized.
4. Budgeting means expressing the plans,
policies and goals of the enterprise for a definite period in future. The
targets are set for different departments and responsibility is fixed for
achieving these targets. The comparison of actual performance with
budgeted figures will give an idea to the management about the
performance of different departments. Forecasting also helps the
management accountant inplanning various activities.
5. Inventory is used to denote stock of raw materials,
goods in process and finished goods. Inventory has a special significance
in accounting for determining correct income for a given period.
Management will need effective inventory control for controlling stocks
and this will helpful for taking managerial decisions.
6. The management accountant interprets
various financial statements to the management. These statements give
an idea about the financial and earning position of the concern. If the
Financial accounting :
Cost Accounting :
Financial Management :
Budgeting and Forecasting :
Inventory Control :
Interpretation of Data :
207
ACCOUNTING FOR MANAGERS
footer
statements are not properly interpreted, these wrong conclusions may be
drawn. So interpretation is as important as compiling of financial
statements.
7. Internal Audit system is necessary to judge the
performance of every department. The actual performance of every
department and individual is compared with pre-determined standards.
Internal audit helps management in fixing responsibility of different
individual.
8. Tax planning is an important part of management
accounting. Income statements are prepared and tax liabilities are
calculated.
Ans. Management Accounting is
comprises of two words Management and Accounting. It is the study of
managerial aspect of accounting. The emphasis of management accounting is
to redesign accounting in such a way that it is helpful to the management in
formation of policy, control of execution and appreciation of effectiveness. It is
that system of accounting which helps management in carrying out its
functions more efficiently. Management Accounting presents the financial data
in such a way as to assist the management in planning and controlling the
activities of the firm. Management Accounting is also known as accounting for
management.
Management Accounting is concerned with accounting information that
is useful to management.
The basic function of
management accounting is to present the information to the management in
such a way that it is helpful to the management in taking correct decisions. The
mainfunctions of the management accounting are:-
1. The first function of management accounting is the
collection of data which is useful to the management. The data is collected
frominternal as well as external sources. Internal sources include profit &
loss account, balance sheet, cost records, sales reports etc. External
sources include business magazines and publication of government
bodies etc.
2. After collection of data, it has to be modified in
such a way that it becomes useful to the management. For example the
productiondata is classified onthe basis of product, quality, time takenby
manufacturing process etc. Data is modified according to the purpose for
whichthe data is required.
Internal Audit :
Tax Accounting :
Q. What are the Functions or Objectives and Limitations of
Management Accounting?
:
:
Robert. N. Anthony
:
Collection of Data :
Modification of Data :
Meaning of Management Accounting
Definitionof Accounting
Functions or Objectives of Management Accounting
208
footer
3. One of the most
significant functions of management accounting is the analysis and
interpretation of profit &loss account and balance sheet of the enterprise.
These are analysed to show the position of liquidity, profitability and
solvency of the enterprise.
4. Management accounting is not
restricted to the use of quantitative informationonly. It collects and makes
use of qualitative informationalso.
5. Planning is a basic function of management and
management accounting provides necessary information to the
management for formulating plans.
6. Organization is the division of duties,
authorities and responsibilities among various persons working in the
enterprise. Management accounting collects necessary data about the
performance of each individual and department and on this basis
evaluates their performance and fixes the responsibility.
7. Management accounting techniques like budgetary
control and standard costing are very helpful in the control process. In
management accounting separate standards are fixed for each
department and actual results are compared with the standards.
Variances are ascertained and taking corrective actions.
8. Management accounting helps in
coordinating the activities of different persons and different departments
by establishing a management informationsystem
9. Management has to make various
decisions. These decisions pertain to introduction of latest technological
devices, selection of product mix, dividend policy etc. Management
accounting helps the management in selecting a suitable alternative and
intaking the best possible decision.
10. One of the main objectives of
management accounting is to provide latest information to various
managers. Informationis provided throughvarious reports.
Although management
accounting has improved the efficiency of management by providing various
service, it suffers fromthe following limitations:-
1.
Management accounting mainly uses the data provided by financial and
cost accounting. These data are historical in nature and are based on
certain accounting assumptions and conventions. Hence, all the
limitations of financial accounting and cost accounting enter in
management accounting.
Analysis and Interpretation of Financial Statements :
Use of Qualitative information also :
To help in planning :
To help in Organizing :
To Help in Control :
Helpful in Coordination :
To help in Decision-Making :
Communicating Up-to-date Information :
:
Limitation of Financial Accounting and Cost Accounting :
Limitations of Management Accounting
209
ACCOUNTING FOR MANAGERS
footer
2. The use of management
accounting requires the knowledge of a number of related subjects such
as accounting, statistics, principles of management and so on. It is very
difficult for the person who is taking the decisions to have a proper
knowledge of all these subjects.
3. Personal views are involved in all the
activities, right from the collection of information till the preparation of
reports submitted to the management. Different persons can draw
different conclusions from the same information. Hence, there is a scope
for bias inmanagement accounting.
4. The installation of management accounting system
requires a large organization and a wide network of rules and regulations
and hence requires a heavy investment. Therefore, smaller organizations
cannot afford it.
5. Management accounting is still passing through
its evolutionary stages and has not yet developed fully.
6. Management accounting is not an
alternative to management. It provides only the informations and not
decisions.
Ans. Management accounting is not a separate method in itself. It is a
combinationof various tools and techniques as follows:-
(1) Ratio Analysis
(2) Funds FlowAnalysis
(3) CashFlowAnalysis
(4) Other techniques of analysis of financial statements such as trend
analysis and comparative financial statements
(5) Budgetary Control
(6) Standard Costing
(7) Marginal Costing
(8) Communicating Or Reporting
(9) Accounting for Price Level Changes
(10) HumanResource Accounting.
Ans. cost accounting provides various techniques for
determining cost of manufacturing products or cost of providing service. It
uses financial data for finding out cost of various jobs, products or process. The
systemof standard costing, marginal costing etc. are all helpful to management
for planning various business activities.
Lack of Knowledge of Related Subjects :
Effected by Personal Views :
Costly System :
Evolutionary Stage :
Not an alternative to Management :
Q. What are the tools and techniques used inmanagement accounting?
Q. DistinguishBetween Cost Accounting and Management Accounting.
Cost Accounting :
210
footer
Management Accounting :
Distinguish between Cost Accounting and Management Accounting:
Sr. Basis of Cost Accounting Management Accounting
No. Difference
Management Accounting presents the financial
data in such a way as to assist the management in planning and controlling the
activities of the firm. Management Accounting is also known as accounting for
management.
1. Objects The main objects of cost The main objects of
accounting are the management accounting
ascertainment of cost, is to provide all types of
controlling the cost and information for
providing the cost data managerial decision
to the management. making, whether derived
from financial accounts or
from cost accounts or
from any other source.
2. Scope The scope of cost Scope of management
accounting is narrow accounting is wider than
because cost accounting cost accounting.
provides information Management accounting
about cost only. includes in its scope
financial accounting, cost
accounting, budgeting,
reporting etc.
3. Nature of Cost accounting is Management accounting
data used related to both past as is concerned only with
well as future. It the future because it
maintains the cost provides information for
records on historical decision making and
basis and also makes planning for the future.
estimates for the future.
4. Use of Cost accounting follows Management accounting
Principles certain principles for does not follow set
recording, classifying principles. It may provide
and ascertaining the information in any
costs of products. manner suitable for
decision making.
5. Use of In cost accounting only Management accounting
Qualitative that data is considered uses both quantitative as
Data which can be expressed well as qualitative data.
211
ACCOUNTING FOR MANAGERS
footer
in quantitative terms, Qualitative data means
non-monetary events like
technological innovation,
change in management,
customer satisfaction,
competition etc.
6 Audit Cost audit has been No audit is required for
made compulsory in management accounting.
certain specified
companies under
section 233 B of the
Companies Act, 1956.
Ans. Management Accounting presents the
financial data in such a way as to assist the management in planning and
controlling the activities of the firm. Management Accounting is also known as
accounting for management.
Financial accounting deals with the historical
data the recorded fact about an organisation are use for planning the future
course of action. Though planning is always for future but still it has to be
based on past and present data. The control aspect too is based on financial
data. The performance appraisal is based on recorded facts and figures. So
management accounting is closely related to financial accounting.
1. Objects The main objective of The main objective of
financial accounting is The main objective
to keep a systematic ofmanagement accounting
record of the is to assist in the internal
transactions of a management of an
business and the enterprise by providing
preparation of profit the necessary
and loss account and information for
the balance sheet. decision making.
Q. Distinguish between Financial Accounting and Management
Accounting.
:
:
Distinguish between Financial Accounting and
Management Accounting:
Sr. Basis of Financial Management
No. Difference
Management Accounting
Financial accounting
Accounting Accounting
212
footer
2. Nature of Financial accounting is Management accounting
Data Used concerned with the is concerned with future.
historical records and
shows only the past
results of a business.
3. Subject Financial accounting Management accounting
Matter measures the measures the profitability
profitability of the entire of different products,
business as a whole. activities, departments or
units separately.
4. Legal Financial accounting is Management accounting
Compulsion compulsory for every is optional.
business because of
legal provisions.
5. Accounting Financial accounts are Management accounting
Principles prepared on the basis of does not follow set
accounting principles principles.
and it has to follow
double entry system.
6. Periodicity Financial statements There is no definite period
are prepared usually for for preparing
one year. management accounts.
7. Accuracy In Financial Accounting In management
only actual figures are accounting since the
recorded. information is needed as
soon as possible, the
approximated figures
which are quickly
available are considered
more useful in comparison
to the accurate figures
which will be available
too late.
8. Monetary Financial accounting In management
Transactions records only those accounting both monetary
transactions which can and non-monetary
be measured in terms information are used.
of money.
213
ACCOUNTING FOR MANAGERS
footer
9. Publications Financial accounts like Management accounts are
profit & loss account prepared for the internal
and balance sheet are use of the management
published for the use of only and hence these are
general public. not published.
10. Audit Financial accounts can Management accounts
be audited and in case cannot be audited since
of companies the they are not based in
financial accounts are actual figures.
required to be audited
by Chartered
Accountants
Ans. Cost accounting is the classifying, recording and
appropriate allocation of expenditure for the determination of the cost of
products or services and for the presentation of suitably arranged data for the
purpose of control of management. Cost accounting includes the calculation of
cost of every product, job, section, department, process, service etc.
Cost accounting is the method of accounting forecast. Cost comprises three
elements viz. material, labour and expense.
According to Wheldon,
Cost Accounting is the application of accounting and costing principles,
methods and techniques in the ascertainment of costs and the analysis of
variances as compared withstandards or previous experience.
1. It provides informationfor cost.
2. It records income and expenditure relating to production of goods and
services.
3. It helps to prepare tenders and quotationprices.
4. It helps incost control.
5. It helps in preparing budgets and standard of cost and the variance
analysis for controlling cost.
6. Cost accounting helps in management in planning, control and decision-
making.
1. Cost knowing or cost finding is cost
Q. Explain the meaning of cost accounting and its importance and
contribution inmanagement.
:
:
:
:
Cost Ascertainment :
Cost Accounting
Definitionof Cost Accounting
Feature of Cost Accounting
Importance of Cost Accounting
214
footer
ascertainment. This is the primary objective of cost accounting. The
technique of ascertaining cost is known as costing. With the help of cost
accounting, cost per unit of a product is calculated.
2. Cost control is one of the important function of cost
accounting. Cost control is reducing the cost of production by controlling
the wastage of material, labour and other expenses. Cost control helps to
improve the efficiency of organization as a whole. For cost control
budgetary control, standard costing are the main tools applied by the
management.
3. It is the object of cost accounting to
ascertain the profitability of the activities carried out. Profitability means
capacity to earn profits of a department, section, and product. This
capacity can be judged with the help of cost data provided by cost
accounting. The expected profits are compared with the actual profits to
know the reasons of difference, if there are any. This helps to analyse the
efficiency of eachsegment of the organizationas well as for the whole of the
organization.
4. The supply price or a tender price of a
product depends upon its total cost. Cost accounting provides detailed
information about the composition of total cost for the determination of
the selling price.
5. Cost accounting helps to provide
accurate cost information. These cost data help the management in taking
short term and long term business policies to be followed. Cost accounts
help in various decision making processes which may be very crucial to
the organization.
6. The objective of cost accounting is to develop
internal audit system which may help in effective working of different
departments of the organization.
7. The objective of cost accounting is to
prepare regular reports regarding material, labour and other expenses
and to communicate those reports at the effective level of management to
make themeffective.
8. Cost accounting helps in making comparison
of cost or of profits of one firms with other firm operating in the same
industry.
1.
Cost accounting provides a systemwhere per unit cost is
calculated. Thus, the management can easily distinguish those products
Cost Control :
Ascertainment of Profitability :
Determination of Selling Price :
Providing a basis for business policy :
Internal Audit System :
Effective Information System :
Inter-firm Comparison :
:
Identification of Profitable and unprofitable activities of the
Organisation :
Contribution of Cost Accounting to Management
215
ACCOUNTING FOR MANAGERS
footer
or departments which are earning profits or those running into lowprofits
or losses.
2. Cost of production can be calculated
on the basis of actual information as well as can be estimated on the basis
of past performance. This helps to quote the price of tenders without
actually performing job.
3. Cost accounting helps to
provide accurate cost information. These cost data help the management
in taking short term and long term business policies to be followed. Cost
accounts help in various decision making processes which may be very
crucial to the organization.
4. Management
has to take vital decision which affects the present and future working of
the organization like make or to buy decision, change in the method of
production, fixation of selling price. All these need various types of cost
data whichare regularly supplied by cost department.
5. There are various techniques of
checking the wastage of material at the time of purchasing, storing and
use of material by various production departments. For this purpose, the
techniques like level of material, ABCanalysis etc. are frequently used.
6. Cost accounts when are maintained
systematically then these may help intra-comparison and inter-firm
comparison regarding cost of production, profitability, fixing of selling
price etc.
7.
(i) Constant efforts are made to reduce the cost of production through
operation, researchtechniques.
(ii) The proper utilization of plants and machines to the full desired
capacity is measured and wastage controlled.
(iii) It helps the management to decide the investment policy.
Ans. Cost has basically three elements, these are:-
1. Material Cost
2. Labour Cost
3. Other Expenses
Calculation of Quotation Price :
Helps in formulating Business Policies :
Helps the management in Decision Making Process :
Helps in Checking Material Cost :
It helps in making comparison :
Other Advantage :
Q. What are the elements of cost. Give the classificationof Costs.
: Elements of Cost
216
footer
Elements of Cost
Material Labour Expenses
Direct Indirect Direct Indirect Direct Indirect
Material Material Labour Labour Expenses Expenses
1. The material may be defined as the item from which
products are manufactured.
(i) Direct materials are those materials which are
either specially purchased for the production of product or which are
visible in the final product, as leather in the shoes, wood in furniture,
clothindress etc.
(ii) These material do not form part of the product.
Indirect material is not visible in the final product as lubricant in
machine, glue inbook binding etc.
2. The term labour may be defined as the human efforts by
whichmaterials are converted into finished products.
(i) Direct labour is the labour which is personally
engaged in the production of goods, running of machines. It is also
knownas direct wages.
(ii) Those workers who are not directly engaged in the
production or running the machines but providing services or help to
those who are operating the machines as storekeeper, watchman,
cleaner, watermanetc.
3. The expenses are incurred for producing a product or
providing a service in addition to material and wages as rent of factory,
insurance , telephone bill, etc.
(i) Direct expenses are those expenses which are
directly identified with a particular job, product or operation.
Direct expenses are also known as process expenses, productive
expenses or prime cost expenses.
(ii) Indirect expenses are those which cannot be
identified witha specific job or process.
Material Cost :
Direct Material :
Indirect Material :
Labour Cost :
Direct Labour :
Indirect Labour :
Expenses :
Direct Expenses :
Indirect Expenses :
217
ACCOUNTING FOR MANAGERS
footer
Classification of Cost :
According to Nature :
According to Function :
Manufacturing Costs :
Administrative Cost :
Selling and Distribution Cost :
Research and Development Cost :
According to Controllability :
Controllable Cost :
Uncontrollable Cost :
Classification of cost means grouping of cost
according to their common characteristics. The main classification of cost are
as follows:-
1. According to Nature or Elements.
2. According to the Functionof Organisation
3. According to Controllability
4. According to Normality
5. According to Accounting Period
6. Others
1. According to this classification, the costs are
divided into three categories.
(i) Material Cost
(ii) Labour Cost
(iii) Other Expenses.
2. According to this classification , the costs are
divided into four categories:-
(i) In any product producing organization,
production is the first activity which generates all other activities of
the organization. It starts with purchase and supply of raw material
to productiondeptt. and ends withthe finished goods kept instore.
(ii) These are the cost incurred for running the
office of the organization where planning and decision-making are
undertaken. These expenses are incurred on the general
management and administrationof the organization.
(iii) The production of goods has no
meaning until and unless there is no demand by the customer. To
create the demand, there is a need of selling and distribution
overheads.
(iv) Research cost are those costs
which are incurred for searching new methods of production or new
product, new material so that it may attract new customers. For all
this, the organization needs better laboratories, specialists staff. This
involves heavy expenditure.
3. Under this category, costs can be divided
into two parts:-
(i) Controllable cost includes those expenses
which can be controlled such as wastage of material, wastage of
power or fuel etc. Generally variable cost are controllable
(ii) It includes those expenses which cannot be
controlled . Generally, fixed costs are uncontrollable.
218
footer
4. As per this classification, the cost can be
divided into:-
(i) Those cost which are expected to incurred on the
production of a product in advance before th production takes place
are normal cost.
(ii) The cost which is incurred in the abnormal cases
or unexpectedly or the cost which was not calculated in advance but
takes place during the productionprocess due to various reasons like
power failure, accident, fire etc.
5. According to this classification, cost
canbe divided into two parts :
(i) Capital Costs are those expenditure the benefits of
which are to be taken over a long enough future period. These
expenditures are very heavy and are to be decided by the top level of
management.
(ii) Those expenditure whichare incurred for day to day
running of the factory. For example purchase of material, payment to
labour, payment of power bill etc.
6.
(i) Standard cost is the pre-determined cost for each
element of production comparing the actual with the standard and
recording the variance, if there is any, for analysis and corrective
action.
(ii) Marginal cost may be defined as the change in the
total cost due to increase or decrease in production by one unit. It
means only the variable costs are considered as the marginal cost.
(iii) It is the current market cost of replacing an
asset.
(iv) Opportunity cost is the value of sacrifice made
or benefit on opportunity foregone in accepting the next best
alternative.
Ans. There are certain techniques for ascertaining
cost which are used by the management in taking managerial decision also.
The major techniques are as follows:-
(1) As the name suggest, under this method first the
expenditure is incurred and then it is recorded and accounted for
calculating cost of production per unit and cost of the product or service.
According to Normality :
Normal Cost :
Abnormal Cost :
According to Accounting Period :
Capital Cost :
Revenue Cost :
Others :
Standard Cost :
Marginal Cost :
Replacement Cost :
Opportunity Cost :
Q. What are the Techniques of Costing?
:
Historical Costing :
Techniques of Costing
219
ACCOUNTING FOR MANAGERS
footer
Methods of Costing
Specific Order Costing Operation Costing
But this method is like a post-mortem of cost of production. This method
cannot applied for measuring efficiency of the organization.
(2) When several firms in the same industry adopt the
same principles of cost accounting for comparison, it is known as uniform
costing.
(3) Marginal cost may be defined as the change in the
total cost due to increase or decrease in production by one unit. It means
only the variable costs are considered as the marginal cost.
(4) Under this system, both variable and fixed cost are
charged to the cost center. Under this technique, both fixed and variable
costs are allocated to the product.
(5) Under this techniques, all costs which are direct to the
product, process whether fixed or variable are charged to cost center.
(6) It is a technique of cost accounting which studies
the change in cost and change in revenue due to change in level of output.
The additional cost and additional revenue are analysed for decision
making by the management.
(7) Standard cost is the pre-determined cost for each
element of production comparing the actual with the standard and
recording the variance, if there is any, for analysis and corrective action.
Ans. Various methods and techniques have been
developed for cost accounting to meet the specific needs of the business
organizations. The methods for calculating cost of production differ from
industry. Basically there are two methods of costing. These are:
A.) Specific Order costing
B.) OperationCosting
A. These methods are applied where the work or
job is of a special nature as the design of the grill, design of house etc. Under
this method production take place when the order is received from the
customer. These include :
Uniform Costing :
Marginal Costing :
Absorption Costing :
Direct Costing :
Incremental Costing :
Standard Costing :
Q. What are the Methods of Costing ORCost Accounting?
:
:
Methods Of Costing
Specific Order costing
220
footer
1. Jobs are to be performed under the specific instructions of
the individual customer. Every customer has special taste, design, quality
of work and spending power. So the jobs are not comparable to each other.
Each job has its own unique features. Job are to be performed in a small
duration, with in the factory premises, and they are generally of small
value. Under job costing, work starts only after receiving the order of the
customer e.g. printing press, tailoring of dress, interior decorators,
machine tolls manufacturing etc.
2. When the small orders of a large number of customers
are made into one group on the basis if similarities if material, method, or
nature of working, it becomes one batch of product to be produced. A
batch may refer to a number of small orders passed through the factory in
a Batch. The cost of each batch is calculated separately and one batch is
considered as one cost unit.
3. This method is applied in those cases where the work
to be performed in big involving lot of investment and take more than one
year to be completed. Contracts are carried outside the factory, generally
at the work site. In contract costing, cost of each contract is ascertained
separately. The contracts are generally of construction nature like
constructionof roads, building, bridges, water work dams etc.
B. When goods are produced on mass scale and of the
standardized nature without waiting for the order of the customer. The
produced goods are kept in stock and the orders of the customers are
complied from the stock of finished goods. The production takes place
continuously. This includes the methods like :
(1) When the raw material has to pass through
various processes, or departments or stages before it is converted into
finished goods, the method applies for the cost calculation in these
industries is known as process costing system. The production is
continuous under this method. In process costing, the output of the first
process becomes the rawmaterial for the second process and the output of
the second process becomes the raw material for the third process and so
on until the product becomes finished goods ready for sale. In process
costing, the cost for production for each unit is calculated at the end of
eachprocess or department.
(2) This method of costing
is applied in those industries which produce identical output and only one
product. The output takes place by continuous operating activities. Cost
per unit is calculated by dividing the total cost by the number of units
produced. Under this method, to calculate the cost of production, a cost
sheet with all the components of cost is prepared for a specific period or
specific order of output.
Job Costing :
Batch Costing :
Contract Costing :
Operating Costing :
Process Costing Method :
Unit Costing Systemor Single output Costing :
221
ACCOUNTING FOR MANAGERS
footer
(3) In cost accounting cost per unit of a
product produced or a service provided is calculated. When cost per unit
of service provided is calculated then the system of costing applied is
knownas operating costing.
Ans. Every enterprise needs inventory for smooth running of its
activities. The terminventory refers to stock of goods kept for sale by the firm.
Kinds of Inventories:-
(A) InTrading Concern. (B) InManufacturing Concern.
(A) In Trading Concern : In case of trading concerns, it includes only finished
goods.
(B) In case of manufacturing concern, inventory
may include:-
(i) Raw Material forma major input into
the organisation. The inventory of raw materials contains the items
which are to be converted into finished goods through the
manufacturing process. The quantity of raw materials required will
be determined by the rate of consumption. The factors like the
availability of raw materials and government regulations, etc. too
affect the stock of rawmaterials.
(ii) The work-in-progress is that stage
of stocks which are in between rawmaterials and finished goods. The
raw materials enter the process of manufacture but they are yet to
attaina final shape of finished goods.
(iii) These are the goods which are ready
for the consumers. In other words, inventory of finished goods
represents completed items whichare available for sale.
1. The valuation of inventory is necessary to
determine the true income earned by a business during a particular
period. Gross profit is the excess of sales over cost of goods sold is
ascertained by adding opening inventory to and deducting closing
inventory formpurchase.
2. The inventory at the end of a
period is to be shown as a current asset in the balance sheet of the
business.
Operating Costing System :
Q. Define Inventory. What are the objectives of Inventory Valuation?
:
Manufacturing Concern :
Inventory of Raw Materials :
Inventory of Work-in-progress :
Inventory of Finished Goods :
:
Determination of Income :
Determination of Financial Position :
Q. What are the methods for Valuation of Inventories?
Inventory
Objectives of Inventory Valuation
222
footer
Ans. The value of materials has a direct bearing on
the income of a concern, so it is necessary that a method of pricing materials
should be such that it gives a realistic value of stocks. The traditional method of
valuing materials Cost price or market price whichever is less is no longer the
only method.
The following methods for pricing materials issues are generally used :
(1) First inFirst Out Method (Knownas FIFOMethod)
(2) Last inFirst Out Method (Knownas LIFOMethod).
(3) Highest inFirst Out (Knownas HIFOMethod).
(4) Average Price Method.
(5) Base Stock Method.
(6) Standard Price Method.
(7) Market Price Method.
(1) In first in first out method the
materials received first are issue first. The materials are issued in
chronological order. The recently received materials remain in stock.
Whenever a requisition for material issue is presented to the store-keeper
he will use the price of the first and thenof second and third lot, etc.
This method is considered more suitable in times when
the prices showthe falling trend. The reason is that higher rate of
material purchased earlier stands recovered in cost and the closing
stock is shownat the current prices.
(i) It is simple to operate.
(ii) It is a logical method as the materials purchased earlier are used
inearlier jobs.
(iii) Stock is valued at the recent purchase prices, and hence closing
stock is valued at the current market price.
(iv) This method is useful whenprices are falling.
(i) In case of fluctuations in prices of materials, calculation
becomes complicated.
(ii) Whenprice fluctuate, there is possibility of clerical errors.
(iii) This method is not suitable whenprices are rising.
Quantity(kg) Rate per Kg Quantity(Kgs.)
Dec.3 200 20
Valuation of Inventories :
First in First Out (FIFO) Method :
Suitable :
Merits of FIFOMethod :
Demerits of FIFOMethod :
Example :
Date Purchases Issues
223
ACCOUNTING FOR MANAGERS
footer
Dec.4 - 100
Dec. 10 - 50
Dec. 18 300 18 -
Dec. 20 - - 300
Dec. 28 50 15 -
Dec. 30 - 100
(2) In last in first out method the last
received materials are issued first and ending inventory consists of earlier
acquired materials. This method is also known as replacement cost
method because the latest purchased goods will correspond to the current
market prices except that goods were not purchased much earlier. The
inventories will be valued at oldest lot on hand and these values will be
quite different fromcurrent invoice prices.
(i) Like LIFOmethod it is simple to operate.
(ii) This method is useful whenprices are rising.
Apply First inFirst Out Method.
Solution :
Stores Ledger Account
Date Receipts Issues Balance
:
Merits of LIFOMethod :
Qty. Rate Amount Qty. Rate Amount Qty. Rate Amount
Dec.3 200 20 4000 ------
Dec.4
Dec.10
Dec.18 300 18 5400
Dec.20
Dec.28 50 15 750
Dec.30
------ ------ 200 20 4000
----- ------ ------ 100 20 2000 100 20 2000
----- ------ ------ 50 20 1000 50 20 1000
------ ------ ------ 50 20 1000
300 18 5400
----- ------ ------ 50 20 1000
250 18 4500 50 18 900
------ ------ ------ 50 18 900
50 15 750
----- ------ ------ 50 18 900
50 15 750 Nil ------ Nil
Last in First Out (LIFO) Method
224
footer
(iii) In this method production is charged at the recent prices because
materials are issued formlatest consignment.
(i) Like FIFOmethod, there are chance of occurring clerical errors.
(ii) Stock are valued at old prices and hence they do not reflect current
prices.
(iii) Under this method materials purchased at last are issued first.
Hence materials purchased earlier become obsolete.
(iv) Closing stock is valued at cost which doesnt represent current
conditions.
Receipts Issues
Date Qty(Tons) Rate(Per Ton) Date Qty (Tons)
Jan1 100 20 Jan4 50
Jan16 300 30 Jan17 200
Jan27 50 50 Jan29 200
Apply Last inFirst Out method.
Solution:
Demerits of LIFOMethod :
:
Stores Ledger Account
Example
Date Receipts Issues Balance
Qty. Rate Amount Qty. Rate Amount Qty. Rate Amount
Jan1 100 20 2000 ------ ------ ------ 100 20 2000
Jan2 ----- ------ ------ 50 20 1000 50 20 2000
Jan16 300 30 9000 ------ ------ ------ 50 20 1000
300 30 9000
Jan17 ----- ------ ------ 200 30 6000 50 20 1000
100 30 3000
Jan27 50 50 2500 ------ ------ ------ 50 20 1000
100 30 3000
50 50 2500
----- ------ ------ 50 50 2500 ------ Nil
Jan29 100 30 3000 Nil
50 20 1000
225
ACCOUNTING FOR MANAGERS
footer
(3) Highest in First Out (HIFO) Method : In this system, the material with
the highest price is issued first. It is based on the assumptions that
stock should be valued at the lowest possible price. The highest priced
materials should be issued first, no matter when they are purchased.
(4) Average Cost Method : In average cost method of pricing all materials in
stock are so mixed that price based on all lots is formed. Average cost may
be of two types:
(a) Simple Average Cost : Inthis method the prices of all lots instock are
averaged and the materials are issued on that average price. For
example, three lots of materials are in stock and the prices per unit
these lots are Rs.2, Rs.3, Rs.4 of first, second and third lots
respectively; thenthe average price will be:
Though this is a simple method of pricing materials but
particularly this method does not give good results. The total cost is
not observed in this method. The following example will explain this
point:
10,000 units were purchased @Rs. 2 per unit
15,000 units were purchased @Rs. 3 per unit
20,000 units were purchased @Rs. 4 per unit
The total cost of materials will be:
10,000 X 2 = 20,000
15,000 X3 = 45,000
20,000 X4 = 80,000
Total Cost = 1, 45,000
The simple average price issue in this case is Rs. 3 and total
amount will become 1,35,000 (45,000X3). The under absorbed
amount in this case will be Rs. 10,000. Because of this weighted
average method is preferred.
(b) In this method the total cost of all the
materials is divided by the total number of items in stock. The price
calculated in this way will be used for issue of materials. Taking the
earlier example the weighed average price will be:
10000 X2 + 15000 X3 + 20000 X4
Weighted Average Price =
10000+ 15000+ 20000
2+3+4
Average Price= = Rs. 3
3
Weighted Average Method :
226
footer
1, 45,000
= = Rs. 3.22
45,000
(5) In this method some quantity of materials is
assumed to be necessary for keeping the concern going. The quantity is
not issued unless otherwise there is an emergency. This material which is
not issued as is kept in stock as a base stock. This method is not an
independent method. It is used alongwith some other methods such as
FIFO, LIFO, Average Price Method, etc. After maintaining the base
quantity in stock, the issues are priced at one of the methods mentioned
above.
(6) The issue price of materials is predetermined
or estimated in this method. The standard price is based on market
conditions, usage rate, storage facilities, etc. The materials are priced at
standard price irrespective of price paid for various purchase.
The Standard price of raw material is fixed at Rs. 5 per
unit. Two lots of materials of 10000 units and 12,000 units were
purchased at Rs. 4.90 and Rs. 5.25 per unit. Every issue of material will be
priced at Rs. 5 per unit, without taking into consideration the prices at
whichthese were purchased.
(7) In this method the price charged to production
are not costs incurred on the materials but latest market prices. It reflects
the latest price charged to production. This method is not generally used
because of a number of difficulties. It becomes difficult to select the
market price because price prevails indifferent markets.
Base Stock Method :
Standard Price Method :
For Example :
Market Price Method :
227
ACCOUNTING FOR MANAGERS
footer
Q. Define Budgetary Control. What are the objectives and Essential
elements of Budgetary Control?
Meaning of Budgetary Control :
:
Essential Elements of Budgetary Control :
:
Mainobjectives :
Planning :
Ans. Budgetary control is applied to system
of management and accounting control by which all operations and outputs
are forecasted so for ahead as possible and actual results when known as
compared withbudget estimate.
According to Chartered Institute of Management Accountants (London)
Budgetary control is the establishment of the budgets relating to the
responsibilities of executives to the requirements of a policy and the
continuous comparison of actual with budgeted results either to so are by
individual action the objectives of that policy or to provide a firm basis for its
revision.
(1) Establishment of Budget for eachfunctionand sectionof the organization.
(2) Continuous comparison of the actual performance with that of the budget
so as to know the variations from budget and placing the responsibility of
executives for failure to achieve the desired results as giveninthe budget
(3) Taking suitable remedial actionto achieve the desired objective if there is a
variationof the actual performance fromthe budgeted performance.
(4) Revisionof budgets inthe light of changed circumstances
The main objectives of budgetary control
are:-
(A)
1. Planning means (a) setting up as objectives (b) setting up of
organization to implement the objectives. Objectives are the targets to be
Definitionof Budgetary Control
Objectives of Budgetary Control
ACCOUNTING FOR MANAGERS
MBA 1st Semester (DDE)
UNIT IV
228
footer
achieved in a particular period by the concern within its limited resources.
The overall planning is represented in the form of master budget which is
prepared withthe help of functional budget.
2. The budgetary control co-ordinates the various
activities of the firm so that the common objective of the firm may be
successfully achieved. It forces executives to think and think as a group. It
also helps inco-ordinating the policies, plans and action.
3. Budgetary control makes control possible by continuous
comparison of actual performance with that of the budget so as report the
variationfromthe budget to the management of corrective action.
(B)
1. Under budgetary control, every department is
given a target to be achieved. The efforts are made to achieve the specific
aims.
2. Budgetary control helps in the formulation the
business policy on general or specific matters like materials, labour,
overheads, sales, productionetc.
3. In budgetary control, various budgets are prepared for
expenditure and then actual are compared with budgeted and taking
corrective actions for deviations.
4. Under budgetary control, the resources are utilized
efficiently in an organization as each person is aware of his task and the
best way by whichit is to be performed.
5. It aims at taking corrective measures. If there is a
difference between budgets and actual results then corrective actions are
taken.
6. The planning of expenditure will be
systematic and there will be economy in spending. The resources are used
to the best advantage.
7. It is a very important tool for effective control because
under it the actual performance is compared with the budgets and
remedial steps are takenincase of deviation, if any.
Ans. Types of Budgets are:
Co-ordination :
Control :
Other Objectives :
Definite Objectives :
Policy Formulation :
Cost Control :
Maximum Profits :
Corrective Measures :
Economy in Expenditure :
Effective Control :
Q. Explain the Different Types of Budgets.
: Types of Budgets
229
ACCOUNTING FOR MANAGERS
footer
Types of Budgets
(A) :
Sales Budget :
According to According to According to Period
Functions Flexibility
1. Sales Budget 1. Fixed Budget 1. Long Period Budget
2. ProductionBudget 2. Flexible Budget 2. Short Period Budget
3. Materials Budget
4. Labour Budget
5. Overhead Budget.
6. CashBudget
7. Master Budget
1. A sale budget is an estimate of expected sales during a
budget period. It is the most important budget and it is called the
backbone of the enterprise. A sale budget is the starting point on which
other budgets are based.
In sale budget expected sales are expressed in quantity as well as in value.
A sales manager is responsible for preparing sales budget. The following
factors should be takeninto account while preparing sales budget:
(i) Past sales figures and facts.
(ii) Availability of RawMaterials
(iii) Seasonal Fluctuations
(iv) Plant Capacity
(v) State of competitioninthe market
(vi) Availability of finance
(vii) Government policy
(viii) Selling price
(ix) Development of Market.
The following informations canbe obtained withthe help of sale budget :
(i) Sales Target
(ii) Possibility of sales indifferent areas.
(iii) What efforts should be made for increasing sales innewareas.?
(iv) Howmuchamount is required to increase the sales?
According to Functions
230
footer
2. Production budget is a forecast of production and
cost of production for a budget period. A production manager is made
responsible for preparing production budget. A production budget is
prepared on the basis of sales budget. The sales budget presents demand
while the production budget makes adequate arrangements for the
fulfillment of this demand. The objective of this budget is to manufacture
the product at minimumcost.
3. Material budget is prepared for determining the
requirement of raw materials for production. This budget depends upon
sales and production budget. The materials are purchased as per the
requirements of productiondepartment.
4. The labour budget required for manufacturing the
product is known as direct labour and the labour which cannot be
specified with production is called indirect labour. This budget is useful
for anticipating labour time required for production.
5. Overheads budget is prepared for the estimation of
indirect expenses related to production i.e. indirect material, indirect
labour and other indirect expenses.
6. Cash budget is a statement of estimates of cash position
for the budget period. It is a plan of estimated receipts and payments of
cash for the budget period. It can be prepared for any time period. Normal
time period of cash budget is half year which is further sub-divided into
the months. It helps in planning and control of the financial requirements
of the organization. In cash budget estimated regarding each item of cash
receipt and payment is made at the time of its preparation:
(i) Cash-Receipts Items :
CashSales
CollectionfromDebtors
Interest Received
Dividend Received
Amount of Tax Refund
Any other income received
(ii) Cash-Payments Items :
Cashpurchase of RawMaterials
Production Budget :
Materials Budget :
Total Material Required = Quantity of material required per unit
XBudgeted Output
Material Cost = Units of Materials required XRate per unit of
RawMaterial
Labour Budget :
Labour Cost = Labour Hours X Rate of pay per hour.
Overhead Budget :
Cash Budget :

231
ACCOUNTING FOR MANAGERS
footer
Payment made to Creditors
Wages, salaries paid
Repayment of Bank Loan
Payment of Taxes
Any other Expenses paid
7. Amaster budget is prepared for the business as a whole,
combining all the budgets for a period into this budget. Thus this budget
gives the overall budget plan for the guidance of the management. This
budget is also known as Summary Budget or the Finalised Profit Plan As
the main objective of budgeting in the profit planning this budget co-
ordinates all the subsidiary budgets in a summary form and shows the
final projected results of the plan.
The following steps are therefore required for preparing a Master Budget :
(i) The preparation of sales budget is the basis starting point for the
preparationof the Master Budget.
(ii) The preparationof the productionbudget is the next step.
(iii) Cost of production budget is the third step in preparation of the
Master Budget.
(iv) The preparationof the cashbudget is the next important step.
(v) The above four steps will be helpful in providing information for
preparing the budgeted or projected income statement.
(vi) On the basis of last years balance sheet and the information
collected by taking above steps, the budgeted or projected balance
sheet for the business will be prepared. This will be the final step in
the preparationof a Master Budget.
1) Fixed Budget is a budget which is desired to remain
unchanged irrespective of the level of activity attained. It does not change
withthe change inlevel of activity actually attained.
2) A flexible budget is a budget designed to change in
accordance with the level of activity actually attained. It varies with the
level of activity attained. Flexible Budget is desirable in the following
cases:
(i) Where the business is newor estimationof demand is not possible.
(ii) Where sales are unpredictable.
(iii) Where the demand for the product keep changing due to change in
fashionand tastes of customers.
(iv) When production cannot be estimated due to irregular supply of
necessary materials and labour.

Master Budget :
(B) :
Fixed Budget :
Flexible Budget :
(C) :
According to Flexibility
According to Period
232
footer
(1) Long period budgets are those budgets which are
prepared for long period.
(2) Short period budgets are those budgets which are
prepared for short period.
Ans. The technique of Zero-base
Budgeting suggests that an organisation should not only make decision about
the proposed new programmes but it should also, fromtime to time, review the
appropriateness of the existing programmes. Such review should particularly
be done of such responsibility centres where there is relatively high proportion
of discretionary costs.
ZBB is a management tool which provides a systematic method for evaluating
all operations and programmes, current or new, allows for budget reductions
and expansions in a rational manner and allows re-allocation of sources from
lowto highpriority programmes.
The process of Zero-base Budgeting involves the following steps :
1. The determination of
the objectives of budgeting is the first step in the system of introducing
Zero-base Budgeting. The objective may be to effect cost reduction in staff
overheads or analyse and drop the projects which do not fit in the
organizational structure or which are not likely to help in achieving the
organizations objectives etc.
2.
This requires going through the organisation chart or
evaluating the pending reorganization or programme realignment. After
studying the organizations structure, the management can decide
whether Zero-base Budgeting is to be introduced in all areas of
organizations activities or only ina fewselected areas ontrial basis.
3. Decision units refer to units regarding
which cost benefit analysis will be done to arrive at a decision whether
they should be allowed to continue or should they be dropped. It may be a
functional department, a programme, a product-line or a sub-line. Each
decision unit must be independent of all the other units so that if the cost
analysis proves unfavorable that unit can be dropped. While selecting
suchdecisionunits, the following points should be kept inmind:
a) They should be capable of being meaningfully reviewed and analyzed.
They should, therefore, neither be too low nor too high in the
organizational hierarchy
Long period Budget :
Short Period Budget :
Q. Write a short note on Zero Base Budgeting (ZBB).
:
:
:
Determination of the Objectives of Budgeting :
Determination of the extent to which the Zero-Base Budgeting is to
be introduced :
Development of Decision Units :
Meaning of Zero-base Budgeting
Definition
Process of Zero-Base Budgeting
233
ACCOUNTING FOR MANAGERS
footer
b) The managers of these decision units should be capable of being
taking significant decisions keeping on view the scope, direction and
quality of work to be performed.
4. This is the most important step
involved in the ZBB process. After identification of decision units, the
manager of each decision unit has to analyse the activities of his own
decision unit or units. He examines the alternative ways of accomplishing
his objectives. He does cost benefit analysis and selects the best
alternative. Then he prepares the decision packages which effectively
summarize his plans and the resources required to achieve them.
5. The decision packages or
(budget requests) after being developed and formulated are submitted to
next level of responsibility within the organisation for ranking purposes.
The objective of suchranking is to put the limited resources at the disposal
of the organisation to the best use. The management ranks the various
decision packages in order of decreasing benefit or importance to the
organisation. The preliminary ranking is done by the decision unit
manager himself who has developed the decision packages. They are then
sent to the superior officers who once again review and rank the decision
packages keeping in view the overall objectives of the organisation in
mind.
6. This is the last stage involved in the ZBB
process. Once the top management has ranked the various decision
packages keeping in view the cost benefit analysis and the availability of
funds, a cut-off point is established. All packages which come within this
cut-off point are accepted and others are rejected. The resources are then
allocated to the different decision units and budgets relating to each unit
are prepared.
Ans. Standard costing is a system of cost accounting
which is designed to find out how much should be the cost of a product under
the existing conditions. The actual cost can be ascertained only when
production is undertaken. The pre-determined cost is compared to the actual
cost and a variance between the two enables the management to take
necessary corrective measures.
(i) The setting of Standards
(ii) Ascertaining actual results.
(iii) Comparing standards and actual costs to determine the variances
(iv) Investigating the variances taking appropriate action where
necessary.
Development of decision Packages :
Review and Ranking of Decision Packages :
Preparation of Budgets :
Q. Define Standard Costing. Explain variance of standard costing. What
are its causes?
:
Standard Costing involves :
Standard Costing
234
footer
Advantage of Standard Costing :
Limitationof Standard Costing :
:
Classificationof Variances :
Material Variance :
Material Cost Variance (MCV) :
Material Cost Variance = (SQXSP) - (AQXAP)
Note :
MATERIAL COST VARIANCE
(i) Measuring efficiency
(ii) Formulationof production&price policy
(iii) Determinationof variance
(iv) Standard cost may result ina reductioninclerical work.
(v) Standard costs are useful indecisionmaking and planning.
(i) Heavy Cost
(ii) Fixationof Responsibility difficult.
(iii) Adverse Psychological Effects
(iv) Unsuitable for Non-Standardized Products
The object of standard costing is to exercise control and cost
reduction. The deviations between standard cost and actual cost will be known
as variances. The variances may be favourable and unfavorable. If actual cost
is less than the standard cost the variances will be favourable and otherwise it
will be adverse or unfavorable.
It may be classified into various categories:
1. Material Variance 2. Labour Variance
3. Overheads Cost Variance 4. Sales or Profit Variance
1. The standard cost of raw material consists of two
elements, quantity and price. Material variance may be classified as
follows :
(i) This represents the difference
betweenthe actual cost and the standard costs of materials.
SQ = Standard Quantity
SP = Standard Price
AQ = Actual Quantity
AP = Actual Price
When there is a difference between standard and actual output
thento calculate (MCV) standard quantities has to revised.
Material Price Material Usage Variance
Material Mix Material Yield
Variance
Variance
Variance Variance
235
ACCOUNTING FOR MANAGERS
footer
(ii) This represents the difference
betweenthe standard price and actual price of materials consumed.
SP = Standard Price
AQ = Actual Quantity
AP = Actual Price
(iii) This represents the differences
between the standard quantity which should have been consumed
and the actual quantity expressed intermof money.
SQ = Standard Quantity
SP = Standard Price
AQ = Actual Quantity
(iv) In many industries it happens that
two or more materials are introduced into a process in a standard
ratio. This is known as a standard mix. The cost of the mix may
therefore differ fromstandard giving rise to a Materials Mix Variance.
When there is a difference between the ratios of mix, only then MMV
arises. It is calculated as follows:
(a) When the ratios of mix is different but the Total Standard
Quantities (TSQ) and the Total Actual Quantities (TAQ) are the
same, in this position the formula of calculating MMV will be as
under :
SQ = Standard Quantity
SP = Standard Price
AQ = Actual Quantity
(b) When the ratio of mix is different and Total Standard Quantities
(TSQ) and the Total Actual Quantities (TAQ) are also different,
then standard quantity of each material will be revised. In this
positionthe formula of calculating MMV will be as under :
RQ = Revised Quantity
SP = Standard Price
AQ = Actual Quantity
TAQ
RQ= SQX
TSQ
Material Price Variance (MPV) :
Material Price Variance = (SP-AP) XAQ
Material Usage Variance (MUV) :
Material Usage Variance= (SQ-AQ) XSP
Material Mix Variance (MMV) :
Material Mix Variance= (SQ-AQ) XSP
Material Mix Variance= (RQ-AQ) XSP
Formula for calculating RQ:
236
footer
If the standard quantity is revised the formula of calculating
Material Usage Variance will be as under:
SQ = Standard Quantity
SP = Standard Price
RQ = Revised Quantity
(v) Material yield variance represents
the portion of material usage variance which is due to the difference
between the standard yield specified and the actual yield obtained.
The actual yield differs fromthe standard yield due to abnormal loss.
It is calculated as under :
(a) When the Total Standard Quantity and Total Actual Quantity
are same, thenthe formula for calculating MYV:
AY = Actual Yield
SY = Standard Yield
SC = Standard Cost per unit
SL = Standard Loss
AL = Actual Loss
Total SCof Standard Mix.
SCPer unit=
SY
(b) When the Total Standard Quantity and Total Actual Quantity
are not same, thenthe formula for calculating MYV:
AY = Actual Yield
RSY = Revised Standard Yield
SC = Standard Cost per unit
Note :
Material Usage Variance= (SQ-RQ) XSP
Material Yield Variance (MYV) :
MYV= (AY SY) XSC
OR
MYV= (SL AL) XSC
MYV= (AY RSY) XSC
OR
MYV= (RSL AL) XSC
Formula for calculating RSY:
237
ACCOUNTING FOR MANAGERS
footer
TAQ
RSY = SY X
TSQ
The relationship between these
material variances canbe expressed as follows :
(i) MCV = MPV+ MUV
(ii) MUV = MMV+ MYV
(iii) MCV = MPV+ MMV+ MYV
(i) Changes inbasic price of materials
(ii) Failures to purchase the quantities anticipated at the time when
standards were set.
(iii) Failure to secure discount onpurchases.
(iv) Failure to make bulk purchases and incurring more onfreight.
(v) Failure to purchase materials at proper time.
(vi) Negligence inuse of materials
(vii) More wastage of materials by untrained workers.
2. There may be two main reasons of the occurrence of
deviations incost of direct labour :
(i) Difference inactual rates and standard rates of labour and
(ii) The variation in the actual time taken by the workers and standard
time allowed to them for performing a job. Labour variances are
classified as follows:-
(i) It is the difference between the standard
labour cost and actual labour cost of the product.
ST = Standard Time
SR = Standard Rate
AT = Actual Time
AR = Actual Rate
(ii) This is also known as LRV. It is that portion of
labour cost variance which is due to the difference between standard rate
specified and the actual rate paid.
(iii) It is that portion of labour cost
variance which arises due to the difference between the standard labour
hours specified and the actual labour hours spent.
Relationship between Material Variances :
Causes of Material Variances :
Labour Variance :
Labour Cost Variance (LCV) :
Labour Cost Variance = (STXSR) - (ATXAR)
Labour Rate Variance :
Labour Rate Variance = (SR-AR) XAT
Labour Efficiency Variance (LEV) :
Labour Efficiency Variance = (ST-AT) XSR
238
footer
(iv) It is that portion of labour efficiency
variance which may arise due to abnormal wastage of time on account of
strikes, power cut, non-availability of raw-material, breakdown of
machinery etc.
(v) Where workers of two or more than two
types are engaged the difference between the standard composition of
workers and the actual gang of workers is known as Labour Mix
Variance. It is calculated as under:-
(a) When Total Actual Time (TAT) spent and Total Standard Time (TST)
are same :
(b) When Total Actual Time (TAT) spent and Total Standard Time (TST)
are not same:
RST = Revised Standard Time
Formula for calculating RST :
TAT
RST = ST X
TST
(vi) It is that portion of labour efficiency
variance which arises due to the difference between actual output of
worker and standard output of worker specified. It is calculated as follows:
(a) When Total Actual Time (TAT) spent and Total Standard Time (TST)
are same :
S
(vii) When Total Actual Time (TAT) spent and Total Standard Time (TST) are not
same :
Formula for calculating Revised Standard Output:
TAT
Revised Standard Output= Standard Output X
TST
Labour Idle Time Variance :
Idle Time Variance = Idle hours XStandard Rate
Labour Mix Variance (LMV) :
Labour Mix Variance = SR(ST-AT)
Labour Mix Variance = SR(RST-AT)
Labour Yield Variance (LYV) :
Labour Yield Variance = (Standard output for actual mix
Actual Output) XSCper unit
Total Cost of Standard Mix
tandard Cost Per Unit=
Net Standard Output
Labour Yield Variance = (Actual Output- Revised Standard
Output) XSc per unit
239
ACCOUNTING FOR MANAGERS
footer
Causes of Labour Variance :
Q. Define Marginal Costing. What are its advantage and disadvantage?
:
Characteristics of Marginal Costing :
:
Helpful in Decision-Making :
Cost Control :
Profit Planning :
Disadvantages of Marginal Costing :
1. Change inbasic wage rate.
2. Excessive Overtime.
3. Use of non-standard material requiring more time to complete work.
4. Defective machinery, tools and equipment.
5. Poor working conditions.
6. Inefficiency of workers.
7. Wrong selectionof workers.
Ans. Marginal cost is the amount of any given volume of
output by which aggregate cost is changed if the volume of output is increased
or decreased by one unit. Marginal Costing is also known as Variable Costing.
In this technique, only variable costs are charged to operation, process or
product.
(1) It is a technique of analysis and presentation of costs which help
management intaking many managerial decisions.
(2) All elements of cost-production, administration and selling and
distributionare classified into variable and fixed components.
(3) Fixed costs are treated as period costs and are charged to Profit &Loss A/c
for the period for whichthey are incurred.
(4) The stock of finished goods and work-in-process are valued at marginal
cost only.
(5) Prices are determined on the basis of marginal costs by adding
contribution whichis the excess of sales.
Advantage of marginal costing are the
following:-
1. Marginal costing plays a significant role in
managerial decision-making process. This systemhelps the management
inplanning, profitability and cost control etc.
2. Under marginal costing all the costs are divided into fixed
cost and variable cost. Variable costs of a product are known as marginal
cost.
3. Marginal cost plays a vital and important role in profit
planning of an organization. Marginal costing: Break-even point, P/V
ratio, marginof safety all help inprofit planning.
While marginal costing & technique is
said to have a number of merits. But there are some demerits also:-
Marginal Costing
Advantages of Marginal Costing
240
footer
1. The long termprice policy, this technique fails to provide solution.
2. This technique is not suitable for external reporting.
Ans. Marginal costing technique is a valuable aid to
management intaking many managerial decisions. It is a useful tool for making
policy decisions, profit planning and cost control. The following are some of the
important managerial problems where marginal costing technique can be
applied.
1. Pricing Decisions
2. Profit Planning and Maintaining a Desired Level of Profit.
3. Make or Buy Decisions.
4. Selectionof a Suitable Sales Mix
5. Effect of Changes inSales Price
6. Alternative Methods of Production
7. Determinationof OptimumLevel of Activity.
8. Evaluationof Performance
9. Capital Investment Decisions
1. Fixing of selling prices is one of the most important
functions of management. Although prices are generally determined by
market conditions and other economic factors yet marginal costing
technique assists the management in the fixation of selling price under
various circumstances as:
(i) Pricing under normal conditions
(ii) During Stiff Competition
(iii) During trade depression
(iv) For accepting special bulk orders
(v) For accepting export orders and exploring newmarkets
2. Marginal
costing techniques can be applied for profit planning as well. Profit
planning involves the planning of future operations to achieve maximum
profits or to maintain a desired level of profits. The change in the sales
price, variable cost and product mix affect the profitability of a concern.
With the help of marginal costing, the required value of sales for
maintaining or attaining a desired level of profit may be ascertained as
follows.
Fixed Cost + Desired Profit
Desired Sales =
P/VRatio
3. Sometimes a concern has to decide whether a
certainproduct should be made in the factory itself or bought fromoutside
froma firmwhichspecializes init. Intaking sucha make or buy decision,
Q. What are the Managerial Applications of Marginal Costing?
:
Pricing Decisions :
Profit Planning and Maintaining a Desired Level of Profit :
Make or Buy Decisions :
Introduction
241
ACCOUNTING FOR MANAGERS
footer
the technique of marginal costing is of immense help. While deciding to
make or buy a distinction must be made between fixed cost and variable
cost, and the variable cost of manufacturing it should be compared with
the price at which this component or product can be bought fromoutside.
It is advisable to make than to but if the variable cost of the product is
lower than the purchase price. But if the purchase price is lower than the
marginal cost, it would be better to buy thanto make itself.
4. When a concern
manufacturers more than one product, a problem often arises as to the
product mix or the sales mix which will yield the maximum profits. In
determining the optimum sales mix, the products which give the
maximum contribution are to be retained and their production should be
increased. The production of products which give comparatively lesser
contribution should be reduced or dropped altogether. Finally the
optimum sales mix is that which gives the highest contribution.
Contributionis calculated as below:
5. Management is generally confronted
with a problem of analyzing the effect of changes in sales price upon the
profitability of the concern. It may be required to reduce the prices on
account of competition, depression, expansion, programme or
government regulations. The effect of changes in sales prices can be easily
analyzed withthe help of contributiontechnique.
6. Sometimes the management has
to choose from among alternative methods of production, e.g., machine
work or hand work. The same product may be produced either by
employing machine No. 1 or Machine No. 2, and the management may be
confronted with the problem of choosing one among them. In such
circumstances, technique of marginal costing can be applied and the
method whichgives the highest contributioncanbe adopted.
7. The technique of
marginal costing also helps the management in determining the optimum
level of activity. To make such a decision, contribution at different levels of
activity can be found, and the level of activity which gives the highest
contribution will be the optimum level. The level of production can be
raised till the marginal cost does not exceed the selling price.
8. Evaluation of performance efficiency of
various department, product lines or markets can also be made with the
use of the technique of marginal costing. Sometimes, the management
may have to decide to discontinue the production of non-profitable
products or departments so as to maximize the profits. In such cases, the
Selection of a Suitable Production/Sales Mix :
Contribution = Sales Variable Cost
Effect of Changes in Sales Price :
Alternative Methods of Production :
Determination of Optimum Level of Activity :
Evaluation of Performance :
242
footer
contribution of different products, departments or sales divisions can be
compared and the one which gives the lowest contribution in comparison
to sales should be discontinued.
9. The technique of marginal costing also
helps the management in taking capital investment decisions. Such
decisions are very crucial for the management and the marginal costing
technique helps the management intaking capital investment decisions.
(A) Profit/Volume Ratio (P/VRatio)
(B) Angle of Incidence
(C) Marginof Safety
(D) Break-evenPoint and Break-EvenChart.
Ans.
The profit volume ratio which is also called the
contribution ratio or marginal ratio express the relation of contribution
to sales and canbe expressed as under :
Contribution= Sales Variable Cost = Fixed Cost + Profit,
Sales Variable Cost S-Vc
(i) P/Vratio = i.e.
Sales S
(ii) Fixed Cost + Profit F + P
P/Vratio = i.e.
Sales S
(iii) Whentwo years data are given:
Change inprofit
P/Vratio= X100
Change inSales
Capital Investment Decision :
Q. Write a short note on:-
(A) Profit / Volume Ratio :
Contribution
P/VRatio = X100
Sales
P/Vratio can also be expressed as :
OR
OR
243
ACCOUNTING FOR MANAGERS
footer
The P/V ratio, which establishes the relationship between contributions on
sales, is of vital importance for studying the profitability of operations of a
business. It reveals the effect on profit of change in the volume. Higher the P/V
ratio, more will be the profit and lower the P/V ratio, lesser will be the profit.
Thus every management aims at increasing the P/V ratio. The ratio can be
increased by increasing the contribution. This canbe done by:-
Increasing the Selling Price per unit.
Reducing the Variable or Marginal Cost.
(B) The angle of incidence is the angle between the
sales line and the total cost line formed at the break-even-point where the
sales line and the total cost line intersect each other. The angle of
incidence indicates profit earning capacity of the business. Alarge angle of
incidence indicates a high rate of profit and on the other hand, a small
angle of incidence indicates a low rate of profit. Usually the angle of
incidence and margin of safety are considered together to indicate the
soundness of a business. A large angle of incidence with a high margin of
safety indicates the most favourable positionof a business.
(C) Margin of safety is the difference between actual sales
and sales at break-even-point. The excess of actual or budgeted sales over
the break-evensales is knownas the marginof safety.
Marginof Safety = Actual sales- Sales at B.E.P.
For Example : If actual sales of a company is Rs. 10,00,000 and the sales at
break-even-point is Rs. 4,00,000 thenmarginof safety is
Marginof safety= 10,00,000 -4,00,000 = 6,00,000
(1) Marginof safety (Inunits) = Actual sales (inunits)- Sales at
B.E.P.(inunits)
(2) Marginof Safety (InRS.) = Actual Sales (InRs.) Sales at
B.E.P. (InRs.)
Profit
(3) Marginof Safety = X 100
P/VRatio
Marginof Safety
(4) Marginof Safety = X100
Actual Sales
The size of margin of safety is an important
indicator of the strength of a business. A business with a greater margin of

Angle of Incidence :
Margin of Safety :
Formulas for calculating Marginof Safety :
: Importance of Margin of Safety
244
footer
safety is more secured and progressive. The margin of safety can be improved
by taking the following steps:-
(i) By increase inthe Selling price.
(ii) By increase the level of production.
(iii) By reducing the fixed cost
(iv) By reducing the variable cost
(v) By substituting unprofitable products withprofitable products
(D) The Break-even point may
be defined as that point of sales volume at which total revenue is equal to
total cost. It is a point of no profits no loss. Abusiness is said to break-even
when its total sales are equal to its total costs. At this point contribution is
equal to fixed costs. If a business is producing more than the one break-
even point there shall be profit to the business organization otherwise it
would suffer a loss. The detailed study of Break-Even Point is known as
Break-EvenAnalysis.
Fixed Cost
(i) Break-evenpoint (Inunits) = -
Contributionper unit
Fixed Cost
(ii) Break-EvenPoint (InRs.) = XSales
Contribution
Fixed Cost
(iii) Break-EvenPoint =
P/VRatio
CashFixed Cost
(iv) CashBreak-evenPoint (Inunits) =
CashContributionper unit
Break-even chart is a graphic presentation of marginal
costing. The break-even chart portrays a pictorial view of the relationships
between costs, volume and profits. The break-even point as indicated in the
chart is the point at whichthe total cost line and the total sales line intersect.
Break-Even-Point and Break-Even Charts :
:
Break Even Chart :
Formulas for calculating Break-Even-Point
245
ACCOUNTING FOR MANAGERS
footer
Example :
Break-Even Chart :
Draw a Break-even Chart
Output VC Total Fixed Total Selling Total
(Per unit) VC Cost Cost price Sales
0 5 0 75000 75000 10 0
5000 5 25000 75000 100000 10 50000
10000 5 50000 75000 125000 10 100000
15000 5 75000 75000 150000 10 150000
20000 5 100000 75000 175000 10 200000
25000 5 125000 75000 200000 10 250000
30000 5 150000 75000 225000 10 300000
300000
275000
250000
225000
200000
175000
150000
125000
100000
75000
50000
25000
0 5000 10000 15000 20000 25000 30000
X
Y
Total Sales Line
Break-Even Point
Angle of Incidence
Total Cost Line
Margin of Safety
Fixed Cost Line
L
o
s
s

A
r
e
a
P
r
o
f
i
t

A
r
e
a
Output (In Units)
246
footer
JULY 2007
Particulars Amount
inRs.
UNITI
UNITII
UNITIII
UNITIV
1. Discuss the nature, functions and significance of accounting.
2. a) What is Profit and Loss Account ? How does it differ from Trading
Account.
b) Mention any three important adjustments that are made for the
preparationof Trading and Profit &Loss Account.
3. What is Fund Flow Statement ? Explain the different sources and
applications of fund.
4. Giventhe following information, work out debt-equity ratio :
Equity Share Capital 6,00,000
Preference Share Capital 2,00,000
General Reserve 2,00,000
Profit &Loss Account
P &L A/CBalance 40,00,000
Profit for the year 2,00,000 6,00,000
13%Convertible Debentures 5,00,000
10 year LoanfromIDBI 3,00,000
Creditors 1,00,000
Provisionfor Tax 2,00,000
Bank Overdraft 1,00,000
5. Management accounting is the presentation of accounting information in
such a way as to assist the management in the creation of policy and in the
day to day operationof the undertaken? Elucidate the above statement.
6. Distinguish between Management Accounting and Cost Accounting. Explain
various classificationof cost inbrief.
7. What is meant by budgetary control ? Discuss the essentials of a good
budgetary control system.
ACCOUNTING FOR MANAGERS
Past Year Question Papers
247
footer
8. The standard cost card reveals the following information:
Labour Rate : Rs. 1/hour
Hours set per unit for production10 hours
Actual data are givenbelow:
Units produced 500
Hours worked 6,000
Actual Labour Cost 4,800
Calculate Labour variances
1. What is a Trial-Balance ? Howit is prepared ? Give examples.
2. Define depreciation. Distinguish between Straight Line and Diminishing
Balance Methods. Give examples.
3. Fromthe following details, prepare a Balance Sheet :
Current Ratio : 1.75
Liquid Ratio : 1.25
Stock Turnover Ratio : 9 times
Gross Profit Ratio : 25%
Debt CollectionPeriod : 1.5 months
Reserves to Capital : 0.2
Turnover of Fixed Assets : 1.2
Capital Gearing Ratio : 0.6
Fixed Assets to Net worth : 1.25
Sales for the year : Rs. 12,00,000
4. State the significance of preparing a funds flow statement. How Funds from
Operations is calculated ?
5. Management Accounting has been evolved to meet the needs of
management. Explainthis statement fully.
6. Why is correct valuation of inventory essential ? Explain LIFO & FIFO
methods of inventories invention.
7. What is budgetary control ? Discuss various advantages and essentials for
the success of budgetary control.
8. Discuss the different marginal costing applications in managerial decision
making.
1. What is the need for providing depreciation ? Discuss with suitable examples
the method of providing depreciationas per Companies Act.
JAN 2007
JULY 2006
UNITI
UNITII
UNITIII
UNITIV
UNITI
248
footer
2. The following is the trial balance of Mr. Arthur as on31 December 2004.
Capital a/c 86,690
Stock on1 Jan. 2004 46,800
Sales 3,89,600
ReturnInwards &Outwards 5,800
Purchases 3,21,700
Freight &Carriage 18,600
Rent and Taxes 5,700
Salary &Wages 9,300
Debtors &Creditors 24,00 14,800
Bank Loan@6%p.a. 20,000
Bank Interest 900
Printing &Advertising 14,600
Misc. Income 250
Cashat Bank 8,000
Discount earned 4,190
Furniture &Fittings 5,000
Discount allowed 1,800
Misc. Exps. 15,9650
CashinHand 380
Drawing 40,000
Following adjustments are to be made :
a) Included amongst the debtors is Rs. 3,000 due fromAnkur and included
among the creditors Rs. 1,000 due to him.
b) Personal purchases amounting to Rs. 600 have been included in
purchase day book.
c) Interest onbank loanis to be provided for the whole year.
d) A quarter of the amount of Printing and Advertising is to be carried
forward to the next year.
e) Stock on31-12-2006 was Rs. 78,600.
Prepare Trading and Profit &Loss Account for the year ended 31-12-2006 and
Balance Sheet as on31-12-2006.
st
st
Dr. Balance Cr. Balance
(Rs.) (Rs.)
5,21,330 5,21,330
249
ACCOUNTING FOR MANAGERS
footer
UNITII
UNITIII
UNITIV
3. Fromthe following particulars extracted fromthe financial statements of ABC
Ltd. Assess the performances over previous year of the company with the help
of relevant ratios and give your comments.
Opening Stock 47,000 53,000
Closing Stock 53,000 67,000
Sales less returns 2,52,000 3,65,000
Provisionfor Bad debts 2,000 3,000
Sundry creditors 32,000 35,000
Purchases 1,80,000 1,90,000
Sundry debtors 42,000 63,000
Cash 10,000 15,000
Bank 15,000 20,000
Bills Receivable 15,000 20,000
Bills payable 29,000 30,000
Marketable Security 8,000 8,000
4. Explaincashflowstatement and its salient features. Also explainits uses.
5. Define Management Accounting. Discuss the techniques, scope and
limitations of management accounting.
6. Briefly discuss, withexamples, the following inventory valuationmethods :
a) First infirst out b) Last infirst out
7. The expenses budgeted for the production of 10,000 units in a factory are as
under :
Materials 70
Labour 25
Variable overheads 20
Fixed overheads (Rs. 1,00,000) 5
Variable expenses 10
Selling expenses (10%Fixed) 15
DistributionExpenses 8
Administrative Expenses (Fixed) 6
Year I Year II
(Rs.) (Rs.)
Per unit (Rs.)
159
250
footer
Prepare a budget for productionof (i) 8,000 units and (ii) 7,000 units.
8. Variance analysis is anintegral part of standard cost accounting. Explain.
1. Prepare a Trial Balance with ten hypothetical transactions. Also show the
journal entries and ledger postings of those accounts.
2. Define depreciation. Show, with an example, how an asset account is
maintained, if the asset is to be disposed off after three years. You may charge
10% per annum depreciation and can use any of the two method, as per
Companys Act.
3. Explainbriefly the meaning and usefulness of the following ratios :
a) Liquidity ratiosb) Profitability ratios
4. The following are the comparative balance sheets of XYZ Ltd. :
Share Capital 70,000 74,000
Debentures 12,000 6,000
Trade Creditors 10,360 11,840
Provisionfor Doubtful debts 700 800
Profits &Loss 10,040 10,560
1,03,200 1,03,200
Cash 9,000 7,800
Trade Debtors (good) 14,900 17,700
Stock-in-trade 49,200 42,700
Land 20,000 30,000
Goodwill 10,000 5,000
1,030,100 1,03,200
Additional Information:
1. Dividends where paid totalling Rs. 3,500
2. Land was purchased for Rs. 10,000 and amount provided for the
amortizationof goodwill totalled Rs. 5,000.
3. Debenture loanwas repaid Rs. 6,000.
Youare required to prepare CashFlowStatement.
JAN 2006
Liabilities 31-12-2003 31-12-2004
(Rs.) (Rs.)
Assets
UNITI
UNITII
251
ACCOUNTING FOR MANAGERS
footer
UNITIII
UNITIV
UNITI
UNITII
UNIT-III
5. Briefly explainwhat are the functions of management accountant.
6. Standard costing is an essential tool of management. Comment on this
statement.
7. What do you mean by Marginal Costing ? Discuss in brief its application in
managerial decisionmaking.
8. Variance analysis is anintegral part of standard cost accounting. Explain.
1. What are the accounting concepts and conventions ? Explain any three of the
following :
i) Dual Aspect Concept
ii) Cost Concept
iii) Conventionof conservation
iv) Business Entity Concept
2. a) Write a short note onTrial Balance.
b) Write a short note onBalance Sheet.
c) Write a short note onDepreciation.
3. A cash flow statement is required to explain change in cash account
balances betweenbalance sheet date. Explainthe statement.
4. From the following particulars, prepare the balance sheet of the form
concerned:
Stock velocity = 6
Capital turnover ratio = 2
Fixed assets turnover ratio = 4
Gross profit = 20%
Debt collectionperiod = 2 months
Creditors payment period = 73 days
The gross profit was Rs. 60,000
Closing stock was Rs. 5,000 inexcess of the Opening stock.
5. Given below are the changes in Account Balance of ABC Ltd. for the year
ending 31 March, 2002.
Rs.
Cash (+) 96,000
Debtors () 16,000
Provisionfor D/D () 400
Stock () 30,000
JULY 2005
st
252
footer
Plant (+) 50,000
Accumulated Depreciation (+) 20,000
Bill payable () 10,000
Outstanding Expenses (+) 800
Rajeevs Capital (+) 89,600
Debtors Rs. 2,000 were written-off as uncollectable. Plant costling Rs. 17,500
was sold for Rs. 7,500 resulting a loss of Rs. 1,000. Net income after charging
the loss onplant amounted to Rs. 1,30,000. Prepare Fund-flowStatement.
6. Define inventory. Why proper valuation of inventory is important ? Also
explainthe LIFOand FIFOmethod of inventory valuation.
7. Define budget and budgetary control. Also discuss the advantages and
limitations of budgetary control system.
8. Due to industrial depression, a plant is running at present, at 50% of its
capacity. The following details are available :
Cost of productionper unit
Direct Material Rs. 2
Direct Labour 1
Variable Overheads 3
Fixed Overhead 2
8
Productionper month 20,000 units
Total cost or production Rs. 1,60,000
Sales Price Rs. 1,40,000
Loss 20,000
An exporter offers to buy 5,000 units per month at the rate of Rs. 6.50 per unit
and the company hesitates to accept the offer for fear of increasing its already
large operating losses. Advise whether offer should be accepted or not.
UNITIV
253
ACCOUNTING FOR MANAGERS
footer
WORKSHEET
254
footer
WORKSHEET
255
ACCOUNTING FOR MANAGERS
footer
WORKSHEET
256
footer

Anda mungkin juga menyukai